Skip to content

The Real and Complex Number Systems

Problem. Prove that there is no largest prime. (A proof was known to Euclid.)

Strategy: Use proof by contradiction. Assume there exists a largest prime pp, then consider N=p!+1N = p! + 1. Since NN is either prime or has a prime factor greater than pp, this contradicts the assumption.

Solution: We will prove this by contradiction. Assume there exists a largest prime number, call it pp.

Consider the number N=p!+1N = p! + 1, where p!p! is the factorial of pp.

Since p!p! is divisible by all integers from 11 to pp, the number N=p!+1N = p! + 1 is not divisible by any prime number less than or equal to pp.

Now, NN is either prime or composite:

  • If NN is prime, then N>pN > p, contradicting our assumption that pp is the largest prime.
  • If NN is composite, then NN has a prime factor qq. Since NN is not divisible by any prime p\leq p, we must have q>pq > p. This again contradicts our assumption that pp is the largest prime.

In both cases, we reach a contradiction. Therefore, our assumption that there exists a largest prime is false, and there must be infinitely many prime numbers.

Problem. If nn is a positive integer, prove the algebraic identity:

anbn=(ab)k=0n1akbn1ka^n - b^n = (a - b)\sum_{k=0}^{n-1} a^k b^{n-1-k}

Strategy: Expand the right-hand side and show it equals the left-hand side by distributing (ab)(a-b) and observing that most terms cancel out.

Solution: We can prove this identity by expanding the right-hand side and showing it equals the left-hand side.

Let’s expand the sum:

(ab)k=0n1akbn1k=(ab)(a0bn1+a1bn2+a2bn3++an1b0)=(ab)(bn1+abn2+a2bn3++an1)\begin{aligned} (a - b)\sum_{k=0}^{n-1} a^k b^{n-1-k} &= (a - b)(a^0 b^{n-1} + a^1 b^{n-2} + a^2 b^{n-3} + \cdots + a^{n-1} b^0) \\ &= (a - b)(b^{n-1} + a b^{n-2} + a^2 b^{n-3} + \cdots + a^{n-1}) \end{aligned}

Now distribute (ab)(a - b):

=abn1+a2bn2+a3bn3++anbbn1abn1a2bn2an1b\begin{aligned} &= a \cdot b^{n-1} + a^2 b^{n-2} + a^3 b^{n-3} + \cdots + a^n \\ &\quad - b \cdot b^{n-1} - a b^{n-1} - a^2 b^{n-2} - \cdots - a^{n-1} b \end{aligned}

Notice that most terms cancel out:

=anbn+(canceling terms)=anbn\begin{aligned} &= a^n - b^n + \text{(canceling terms)} \\ &= a^n - b^n \end{aligned}

Alternatively, we can use the geometric series formula. Let r=abr = \frac{a}{b}. Then:

k=0n1akbn1k=bn1k=0n1(ab)k=bn11(ab)n1ab=bn1bnanbn(ba)=anbnab\begin{aligned} \sum_{k=0}^{n-1} a^k b^{n-1-k} &= b^{n-1} \sum_{k=0}^{n-1} \left(\frac{a}{b}\right)^k \\ &= b^{n-1} \cdot \frac{1 - \left(\frac{a}{b}\right)^n}{1 - \frac{a}{b}} \\ &= b^{n-1} \cdot \frac{b^n - a^n}{b^n(b - a)} \\ &= \frac{a^n - b^n}{a - b} \end{aligned}

Therefore, (ab)k=0n1akbn1k=anbn(a - b)\sum_{k=0}^{n-1} a^k b^{n-1-k} = a^n - b^n.

Problem. If 2n12^n - 1 is prime, prove that nn is prime. A prime of the form 2p12^p - 1, where pp is prime, is called a Mersenne prime.

Strategy: Prove the contrapositive: if nn is composite, then 2n12^n - 1 is composite. Use the identity from Problem 1.2 to factor 2n12^n - 1 when n=abn = ab with a,b>1a, b > 1.

Solution: We will prove the contrapositive: if nn is composite, then 2n12^n - 1 is composite.

Let n=abn = ab where a,b>1a, b > 1 are integers. Then:

2n1=2ab1=(2a)b1\begin{aligned} 2^n - 1 &= 2^{ab} - 1 \\ &= (2^a)^b - 1 \end{aligned}

Using the identity from Problem 1.2 with x=2ax = 2^a and y=1y = 1:

(2a)b1=(2a1)((2a)b1+(2a)b2++2a+1)\begin{aligned} (2^a)^b - 1 &= (2^a - 1)((2^a)^{b-1} + (2^a)^{b-2} + \cdots + 2^a + 1) \end{aligned}

Since a>1a > 1, we have 2a1>12^a - 1 > 1. Also, since b>1b > 1, the second factor is greater than 1. Therefore, 2n12^n - 1 is the product of two integers greater than 1, making it composite.

This proves that if 2n12^n - 1 is prime, then nn must be prime.

Problem. If 2n+12^n + 1 is prime, prove that nn is a power of 22. A prime of the form 22n+12^{2^n} + 1 is called a Fermat prime. Hint: Use Exercise 1.2.

Strategy: Prove the contrapositive: if nn is not a power of 22, then 2n+12^n + 1 is composite. When nn has an odd factor, use the identity from Problem 1.2 to factor 2n+12^n + 1.

Solution: We will prove the contrapositive: if nn is not a power of 22, then 2n+12^n + 1 is composite.

If nn is not a power of 22, then nn has an odd factor greater than 11. Let n=2kmn = 2^k \cdot m where m>1m > 1 is odd and k0k \geq 0.

Then:

2n+1=22km+1=(22k)m+1\begin{aligned} 2^n + 1 &= 2^{2^k \cdot m} + 1 \\ &= (2^{2^k})^m + 1 \end{aligned}

Since mm is odd, we can use the identity from Problem 1.2 with a=22ka = 2^{2^k} and b=1b = -1:

(22k)m(1)m=(22k(1))((22k)m1+(22k)m2(1)++(1)m1)\begin{aligned} (2^{2^k})^m - (-1)^m &= (2^{2^k} - (-1))((2^{2^k})^{m-1} + (2^{2^k})^{m-2}(-1) + \cdots + (-1)^{m-1}) \end{aligned}

Since mm is odd, (1)m=1(-1)^m = -1, so:

(22k)m+1=(22k+1)((22k)m1(22k)m2++1)\begin{aligned} (2^{2^k})^m + 1 &= (2^{2^k} + 1)((2^{2^k})^{m-1} - (2^{2^k})^{m-2} + \cdots + 1) \end{aligned}

Since m>1m > 1, both factors are greater than 11, making 2n+12^n + 1 composite.

Therefore, if 2n+12^n + 1 is prime, then nn must be a power of 22.

Problem. The Fibonacci numbers 1,1,2,3,5,8,13,1, 1, 2, 3, 5, 8, 13, \dots are defined by the recursion formula xn+1=xn+xn1x_{n+1} = x_n + x_{n-1}, with x1=x2=1x_1 = x_2 = 1. Prove that xn=anbnabx_n = \frac{a^n - b^n}{a - b}, where aa and bb are the roots of the equation x2x1=0x^2 - x - 1 = 0.

Strategy: Use strong mathematical induction. Verify base cases for n=1n=1 and n=2n=2, then use the inductive hypothesis and the key property that a2=a+1a^2 = a+1 and b2=b+1b^2 = b+1 to establish the inductive step.

Solution: Let the proposition be P(n):xn=anbnabP(n): x_n = \frac{a^n - b^n}{a-b}. The roots of x2x1=0x^2 - x - 1 = 0 are a=1+52a = \frac{1+\sqrt{5}}{2} and b=152b = \frac{1-\sqrt{5}}{2}. A key property of these roots is that they satisfy a2=a+1a^2 = a+1 and b2=b+1b^2 = b+1.

Base cases: For n=1n=1:

a1b1ab=1=x1.\frac{a^1 - b^1}{a-b} = 1 = x_1.

For n=2n=2:

a2b2ab=(ab)(a+b)ab=a+b=(1+52)+(152)=1=x2.\begin{aligned} \frac{a^2 - b^2}{a-b} &= \frac{(a-b)(a+b)}{a-b}\\ &= a+b\\ &= \left(\frac{1+\sqrt{5}}{2}\right) + \left(\frac{1-\sqrt{5}}{2}\right)\\ &= 1 = x_2. \end{aligned}

The base cases hold.

Inductive step: Assume P(k)P(k) is true for all integers knk \leq n, where n2n \geq 2. We will show P(n+1)P(n+1) is true. By the definition of the Fibonacci sequence, xn+1=xn+xn1x_{n+1} = x_n + x_{n-1}. Using the inductive hypothesis for xnx_n and xn1x_{n-1}:

xn+1=(anbnab)+(an1bn1ab)=(an+an1)(bn+bn1)ab=an1(a+1)bn1(b+1)ab\begin{aligned} x_{n+1} &= \left( \frac{a^n - b^n}{a-b} \right) + \left( \frac{a^{n-1} - b^{n-1}}{a-b} \right) \\ &= \frac{(a^n + a^{n-1}) - (b^n + b^{n-1})}{a-b} \\ &= \frac{a^{n-1}(a+1) - b^{n-1}(b+1)}{a-b} \end{aligned}

Using the property that a+1=a2a+1 = a^2 and b+1=b2b+1 = b^2:

xn+1=an1(a2)bn1(b2)ab=an+1bn+1ab\begin{aligned} x_{n+1} &= \frac{a^{n-1}(a^2) - b^{n-1}(b^2)}{a-b} \\ &= \frac{a^{n+1} - b^{n+1}}{a-b} \end{aligned}

This is P(n+1)P(n+1). By the principle of strong induction, the formula is true for all positive integers nn.

Problem. Prove that every nonempty set of positive integers contains a smallest member. This is called the well-ordering principle.

Strategy: Use proof by contradiction combined with mathematical induction. Assume there exists a nonempty set with no smallest member, then use induction to show this leads to the set being empty.

Solution: We will prove this by contradiction. Let SS be a nonempty set of positive integers that has no smallest member. Let P(n)P(n) be the proposition that the integer nn is not in SS. We will use induction to show that P(n)P(n) is true for all positive integers nn.

Base case: For n=1n=1: If 1S1 \in S, then 11 would be the smallest member of SS (since SS contains only positive integers). But we assumed SS has no smallest member. So 11 cannot be in SS. Thus, P(1)P(1) is true.

Inductive step: Assume that P(k)P(k) is true for all positive integers k<nk < n. This means that none of the integers 1,2,,n11, 2, \dots, n-1 are in SS. Now consider the integer nn. If nSn \in S, then from our inductive hypothesis (that 1,2,,n11, 2, \dots, n-1 are not in SS), nn would be the smallest member of SS. This contradicts our initial assumption that SS has no smallest member. Therefore, nn cannot be in SS. Thus, P(n)P(n) is true.

By the principle of strong induction, P(n)P(n) is true for all positive integers nn. This means that no positive integer is in SS, which implies that SS is an empty set. This contradicts our initial assumption that SS is a nonempty set. Therefore, the original assumption must be false, and every nonempty set of positive integers must contain a smallest member.

Theorem (Geometric Series Formula).

For r<1|r| < 1, the infinite geometric series converges:

n=0arn=a1r\sum_{n=0}^{\infty} ar^n = \frac{a}{1-r}

For a finite geometric series:

n=0N1arn=a1rN1r\sum_{n=0}^{N-1} ar^n = a \cdot \frac{1-r^N}{1-r}

Importance: The geometric series is one of the most fundamental infinite series in mathematics. It provides the foundation for understanding power series, Taylor series, and many other important series expansions. This formula is essential for calculus, analysis, and many applications in physics and engineering.

Theorem (Terminating Decimal Criterion).

A rational number pq\frac{p}{q} has a terminating decimal expansion if and only if the prime factorization of qq contains only powers of 22 and 55.

Importance: This theorem provides a precise criterion for determining when a rational number has a finite decimal representation. It connects number theory (prime factorization) to decimal arithmetic, making it easy to predict the behavior of decimal expansions. This result is essential for understanding decimal representations and their applications.

Theorem (Irrationality of Square Roots).

If nn is a positive integer that is not a perfect square, then n\sqrt{n} is irrational.

Importance: This theorem provides a large class of irrational numbers and is fundamental for understanding the relationship between algebra and geometry. It shows that many naturally occurring numbers (like 2\sqrt{2}, 3\sqrt{3}, etc.) are irrational, which is essential for geometry, algebra, and many areas of mathematics. The proof technique used here is a classic example of proof by contradiction.

Theorem (Density of Rationals and Irrationals).

Between any two real numbers, there exist both rational and irrational numbers.

Importance: This theorem shows that both rational and irrational numbers are densely distributed throughout the real number line. This property is fundamental for understanding the structure of the real numbers and is essential for many results in analysis, topology, and approximation theory. It demonstrates that the real number system is rich and complex.

Problem 1.7: Decimal Expansion to Rational

Section titled “Problem 1.7: Decimal Expansion to Rational”

Problem. Find the rational number whose decimal expansion is 0.3344440.334444\ldots.

Strategy: Use an algebraic method by multiplying by powers of 10 to shift the decimal point and eliminate the repeating part, then solve for the unknown fraction.

Solution: We can use an algebraic method to find the equivalent fraction. Let xx be the rational number. x=0.334444x = 0.334444\ldots The goal is to manipulate the equation to eliminate the repeating decimal part. The repeating digit ‘4’ begins at the third decimal place.

First, multiply by 100100 to move the non-repeating part to the left of the decimal point: 100x=33.4444100x = 33.4444\ldots Next, multiply by 10001000 to shift the decimal point past the first repeating digit: 1000x=334.44441000x = 334.4444\ldots Now, subtract the first equation from the second. This will cancel the infinite repeating tail.

1000x=334.4444100x=033.4444900x=301\begin{aligned} 1000x &= 334.4444\ldots \\ -\quad 100x &= \phantom{0}33.4444\ldots \\ \hline 900x &= 301 \end{aligned}

Finally, solve for xx: x=301900x = \frac{301}{900} Therefore, the rational number is **\frac{301**{900}}.

Problem 1.8: Decimal Expansion Ending in Zeroes

Section titled “Problem 1.8: Decimal Expansion Ending in Zeroes”

Problem. Prove that the decimal expansion of xx will end in zeroes (or in nines) if and only if xx is a rational number whose denominator is of the form 2m5n2^m 5^n, where mm and nn are nonnegative integers.

Strategy: Prove both directions of the if-and-only-if statement. Show that rational numbers with denominators of the form 2m5n2^m 5^n have terminating decimal expansions, and conversely that terminating decimals correspond to such rational numbers.

Solution: We need to prove both directions of this statement.

Forward direction: If xx is rational with denominator of the form 2m5n2^m 5^n, then its decimal expansion terminates.

Let x=pqx = \frac{p}{q} where q=2m5nq = 2^m 5^n for some nonnegative integers m,nm, n.

We can write x=p2m5n=p2n5m2m5n2n5m=p2n5m10m+nx = \frac{p}{2^m 5^n} = \frac{p \cdot 2^n 5^m}{2^m 5^n \cdot 2^n 5^m} = \frac{p \cdot 2^n 5^m}{10^{m+n}}

This shows that xx can be written as a fraction with denominator a power of 10, which means its decimal expansion terminates.

Reverse direction: If the decimal expansion of xx terminates, then xx is rational with denominator of the form 2m5n2^m 5^n.

Let xx have a terminating decimal expansion. Then xx can be written as x=N10kx = \frac{N}{10^k} for some integer NN and nonnegative integer kk.

Since 10=2510 = 2 \cdot 5, we have 10k=2k5k10^k = 2^k \cdot 5^k, which is of the required form.

Note about ending in nines: If a decimal expansion ends in nines (e.g., 0.9990.999\ldots), this is equivalent to the next terminating decimal. For example, 0.999=1.0000.999\ldots = 1.000\ldots. This is because 0.999=910+9100+91000+=9/1011/10=10.999\ldots = \frac{9}{10} + \frac{9}{100} + \frac{9}{1000} + \cdots = \frac{9/10}{1 - 1/10} = 1.

Therefore, both terminating decimals and those ending in nines correspond to rational numbers with denominators of the form 2m5n2^m 5^n.

Problem 1.9: Irrationality of 2+3\sqrt{2} + \sqrt{3}

Section titled “Problem 1.9: Irrationality of 2+3\sqrt{2} + \sqrt{3}2​+3​”

Problem. Prove that 2+3\sqrt{2} + \sqrt{3} is irrational.

Strategy: Use proof by contradiction. Assume 2+3\sqrt{2} + \sqrt{3} is rational, then square both sides to eliminate the square roots. This leads to 6\sqrt{6} being rational, which is false.

Solution: We will prove this by contradiction. Assume that 2+3\sqrt{2} + \sqrt{3} is rational, say 2+3=pq\sqrt{2} + \sqrt{3} = \frac{p}{q} where p,qp, q are integers with no common factors.

Then:

2+3=pq(2+3)2=(pq)22+26+3=p2q25+26=p2q226=p2q256=p25q22q2\begin{aligned} \sqrt{2} + \sqrt{3} &= \frac{p}{q} \\ (\sqrt{2} + \sqrt{3})^2 &= \left(\frac{p}{q}\right)^2 \\ 2 + 2\sqrt{6} + 3 &= \frac{p^2}{q^2} \\ 5 + 2\sqrt{6} &= \frac{p^2}{q^2} \\ 2\sqrt{6} &= \frac{p^2}{q^2} - 5 \\ \sqrt{6} &= \frac{p^2 - 5q^2}{2q^2} \end{aligned}

This shows that 6\sqrt{6} is rational, which is a contradiction since 6\sqrt{6} is irrational.

To see why 6\sqrt{6} is irrational, suppose 6=ab\sqrt{6} = \frac{a}{b} where a,ba, b are integers with no common factors. Then:

6=a2b26b2=a2\begin{aligned} 6 &= \frac{a^2}{b^2} \\ 6b^2 &= a^2 \end{aligned}

This means a2a^2 is divisible by 6, so aa must be divisible by 6. Let a=6ka = 6k. Then:

6b2=(6k)2=36k2b2=6k2\begin{aligned} 6b^2 &= (6k)^2 = 36k^2 \\ b^2 &= 6k^2 \end{aligned}

This means b2b^2 is divisible by 6, so bb must also be divisible by 6. But this contradicts our assumption that aa and bb have no common factors.

Therefore, 6\sqrt{6} is irrational, and consequently 2+3\sqrt{2} + \sqrt{3} is irrational.

Problem 1.10: Rational Functions of Irrational Numbers

Section titled “Problem 1.10: Rational Functions of Irrational Numbers”

Problem. If a,b,c,da, b, c, d are rational and if xx is irrational, prove that ax+bcx+d\frac{ax + b}{cx + d} is usually irrational. When do exceptions occur?

Strategy: Assume the expression is rational and solve for the conditions under which this can happen. Use the fact that if a rational expression equals a rational number, then the coefficients must satisfy certain relationships.

Solution: We need to analyze when ax+bcx+d\frac{ax + b}{cx + d} is rational, given that xx is irrational and a,b,c,da, b, c, d are rational.

Let’s assume that ax+bcx+d=pq\frac{ax + b}{cx + d} = \frac{p}{q} where p,qp, q are integers with no common factors.

Then:

ax+bcx+d=pqq(ax+b)=p(cx+d)qax+qb=pcx+pd(qapc)x=pdqb\begin{aligned} \frac{ax + b}{cx + d} &= \frac{p}{q} \\ q(ax + b) &= p(cx + d) \\ qax + qb &= pcx + pd \\ (qa - pc)x &= pd - qb \end{aligned}

Since xx is irrational and the right-hand side is rational, we must have qapc=0qa - pc = 0 and pdqb=0pd - qb = 0.

This gives us:

qa=pcpd=qb\begin{aligned} qa &= pc \\ pd &= qb \end{aligned}

From the first equation: a=pcqa = \frac{pc}{q} From the second equation: b=pdqb = \frac{pd}{q}

Therefore, we have:

ac=pqbd=pq\begin{aligned} \frac{a}{c} &= \frac{p}{q} \\ \frac{b}{d} &= \frac{p}{q} \end{aligned}

This means ac=bd\frac{a}{c} = \frac{b}{d}, or equivalently, ad=bcad = bc.

Conclusion: The expression ax+bcx+d\frac{ax + b}{cx + d} is rational if and only if ad=bcad = bc.

Exceptions occur when: ad=bcad = bc, which means the numerator and denominator are proportional, making the fraction rational regardless of the value of xx.

Examples:

  • If a=2,b=1,c=4,d=2a = 2, b = 1, c = 4, d = 2, then ad=4=bc=4ad = 4 = bc = 4, so 2x+14x+2=12\frac{2x + 1}{4x + 2} = \frac{1}{2} for all xx.
  • If a=1,b=0,c=1,d=0a = 1, b = 0, c = 1, d = 0, then ad=0=bc=0ad = 0 = bc = 0, so xx=1\frac{x}{x} = 1 for all x0x \neq 0.

Problem 1.11: Irrational Numbers Between 0 and x

Section titled “Problem 1.11: Irrational Numbers Between 0 and x”

Problem. Given any real x>0x > 0, prove that there is an irrational number between 00 and xx.

Strategy: Construct an irrational number between 00 and xx by considering two cases: when xx is irrational (use x2\frac{x}{2}) and when xx is rational (use x2\frac{x}{\sqrt{2}}).

Solution: We will construct an irrational number between 00 and xx for any positive real number xx.

Case 1: If xx is irrational, then x2\frac{x}{2} is irrational and lies between 00 and xx.

To see why x2\frac{x}{2} is irrational, suppose it were rational. Then x2=pq\frac{x}{2} = \frac{p}{q} for some integers p,qp, q, which would mean x=2pqx = \frac{2p}{q}, making xx rational, a contradiction.

Case 2: If xx is rational, let x=pqx = \frac{p}{q} where p,qp, q are positive integers.

Consider the number y=x2=pq2y = \frac{x}{\sqrt{2}} = \frac{p}{q\sqrt{2}}.

Since 2\sqrt{2} is irrational, yy is irrational (if yy were rational, then 2=pqy\sqrt{2} = \frac{p}{qy} would be rational, a contradiction).

Also, since 2>1\sqrt{2} > 1, we have y<xy < x.

Therefore, yy is an irrational number between 00 and xx.

Alternative construction: For any positive real xx, we can also use y=xπy = \frac{x}{\pi}. Since π\pi is irrational and greater than 11, we have 0<y<x0 < y < x, and yy is irrational.

Problem 1.12: Fraction Between Two Fractions

Section titled “Problem 1.12: Fraction Between Two Fractions”

Problem. If ab<cd\frac{a}{b} < \frac{c}{d} with b>0,d>0b > 0, d > 0, prove that a+cb+d\frac{a + c}{b + d} lies between ab\frac{a}{b} and cd\frac{c}{d}.

Strategy: Prove both inequalities ab<a+cb+d\frac{a}{b} < \frac{a + c}{b + d} and a+cb+d<cd\frac{a + c}{b + d} < \frac{c}{d} by cross-multiplying and using the given condition ab<cd\frac{a}{b} < \frac{c}{d}.

Solution: We need to prove that ab<a+cb+d<cd\frac{a}{b} < \frac{a + c}{b + d} < \frac{c}{d}.

Let’s prove both inequalities:

First inequality: ab<a+cb+d\frac{a}{b} < \frac{a + c}{b + d}

Cross-multiplying:

a(b+d)<b(a+c)ab+ad<ab+bcad<bc\begin{aligned} a(b + d) &< b(a + c) \\ ab + ad &< ab + bc \\ ad &< bc \end{aligned}

Since ab<cd\frac{a}{b} < \frac{c}{d}, we have ad<bcad < bc, so this inequality holds.

Second inequality: a+cb+d<cd\frac{a + c}{b + d} < \frac{c}{d}

Cross-multiplying:

d(a+c)<c(b+d)ad+cd<bc+cdad<bc\begin{aligned} d(a + c) &< c(b + d) \\ ad + cd &< bc + cd \\ ad &< bc \end{aligned}

Again, since ab<cd\frac{a}{b} < \frac{c}{d}, we have ad<bcad < bc, so this inequality also holds.

Therefore, a+cb+d\frac{a + c}{b + d} lies between ab\frac{a}{b} and cd\frac{c}{d}.

Geometric interpretation: This result is known as the “mediant” of two fractions. If we think of fractions as points on a line, the mediant a+cb+d\frac{a + c}{b + d} lies between the two original fractions ab\frac{a}{b} and cd\frac{c}{d}.

Problem 1.13: 2\sqrt{2} Between Fractions

Section titled “Problem 1.13: 2\sqrt{2}2​ Between Fractions”

Problem. Let aa and bb be positive integers. Prove that 2\sqrt{2} always lies between the two fractions ab\frac{a}{b} and a+2ba+b\frac{a + 2b}{a + b}. Which fraction is closer to 2\sqrt{2}?

Strategy: Analyze the relationship between the two fractions and 2\sqrt{2} by examining their difference. Consider two cases based on whether ab\frac{a}{b} is less than or greater than 2\sqrt{2}, then compare the distances to determine which fraction is closer.

Solution: Let’s first establish the ordering of the two fractions by examining their difference:

a+2ba+bab=b(a+2b)a(a+b)b(a+b)=ab+2b2a2abb(a+b)=2b2a2b(a+b)\frac{a + 2b}{a + b} - \frac{a}{b} = \frac{b(a + 2b) - a(a + b)}{b(a + b)} = \frac{ab + 2b^2 - a^2 - ab}{b(a + b)} = \frac{2b^2 - a^2}{b(a + b)}

The sign of this difference depends on the sign of 2b2a22b^2 - a^2, which relates ab\frac{a}{b} to 2\sqrt{2}.

**Case 1: \frac{a**{b} < \sqrt{2}}. This means a<b2a < b\sqrt{2}, so a2<2b2a^2 < 2b^2, and 2b2a2>02b^2 - a^2 > 0. Thus, ab<a+2ba+b\frac{a}{b} < \frac{a+2b}{a+b}. We need to show that a+2ba+b>2\frac{a+2b}{a+b} > \sqrt{2}.

a+2ba+b>2    a+2b>2(a+b)    b(22)>a(21)    2221>ab\begin{aligned} \frac{a + 2b}{a + b} > \sqrt{2} &\iff a + 2b > \sqrt{2}(a + b)\\ &\iff b(2 - \sqrt{2}) > a(\sqrt{2} - 1)\\ &\iff \frac{2 - \sqrt{2}}{\sqrt{2} - 1} > \frac{a}{b} \end{aligned}

Since 2221=2(21)21=2\frac{2 - \sqrt{2}}{\sqrt{2} - 1} = \frac{\sqrt{2}(\sqrt{2} - 1)}{\sqrt{2} - 1} = \sqrt{2}, this simplifies to 2>ab\sqrt{2} > \frac{a}{b}, which is true by our case assumption. Thus, ab<2<a+2ba+b\frac{a}{b} < \sqrt{2} < \frac{a + 2b}{a + b}.

**Case 2: \frac{a**{b} > \sqrt{2}}. This means a2>2b2a^2 > 2b^2, and 2b2a2<02b^2 - a^2 < 0. Thus, ab>a+2ba+b\frac{a}{b} > \frac{a+2b}{a+b}. A similar calculation shows that a+2ba+b<2\frac{a+2b}{a+b} < \sqrt{2}. Therefore, a+2ba+b<2<ab\frac{a+2b}{a+b} < \sqrt{2} < \frac{a}{b}. In both cases, 2\sqrt{2} lies between the two fractions.

**Which fraction is closer to \sqrt{2**?} We compare the absolute distances:

  • Distance 1: ab2=ab2b\left|\frac{a}{b} - \sqrt{2}\right| = \frac{|a - b\sqrt{2}|}{b}
  • Distance 2: a+2ba+b2=a+2ba2b2a+b=a(12)b(22)a+b=ab2(21)a+b\left|\frac{a + 2b}{a + b} - \sqrt{2}\right| = \left|\frac{a + 2b - a\sqrt{2} - b\sqrt{2}}{a + b}\right| = \left|\frac{a(1-\sqrt{2}) - b(\sqrt{2}-2)}{a + b}\right| = \frac{|a - b\sqrt{2}|(\sqrt{2}-1)}{a+b}

To see which distance is smaller, we compare 1b\frac{1}{b} with 21a+b\frac{\sqrt{2}-1}{a+b}. This is equivalent to comparing a+ba+b with b(21)=b2bb(\sqrt{2}-1) = b\sqrt{2} - b, which is equivalent to comparing a+2ba+2b with b2b\sqrt{2}, or ab+2\frac{a}{b} + 2 with 2\sqrt{2}. Since a,ba, b are positive integers, ab>0\frac{a}{b} > 0, so ab+2>2>2\frac{a}{b} + 2 > 2 > \sqrt{2}. This implies 1b>21a+b\frac{1}{b} > \frac{\sqrt{2}-1}{a+b}. Therefore, Distance 1 is always greater than Distance 2. The new fraction a+2ba+b\frac{a+2b}{a+b} is always closer to 2\sqrt{2}.

Visualization:

[TikZ diagram omitted in web version]

This visualization shows how the process of generating new fractions a+2ba+b\frac{a+2b}{a+b} from ab\frac{a}{b} always produces a better approximation to 2\sqrt{2}. The red dashed line represents 2\sqrt{2}, and the arrows show the improvement in approximation.

Problem 1.14: Irrationality of n1+n+1\sqrt{n - 1} + \sqrt{n + 1}

Section titled “Problem 1.14: Irrationality of n−1+n+1\sqrt{n - 1} + \sqrt{n + 1}n−1​+n+1​”

Problem. Prove that n1+n+1\sqrt{n - 1} + \sqrt{n + 1} is irrational for every integer n1n \geq 1.

Strategy: Use proof by contradiction. Assume the sum is rational, then square both sides to eliminate the square roots. This leads to n21\sqrt{n^2 - 1} being rational, which is false since n21n^2 - 1 is not a perfect square for n2n \geq 2.

Solution: Assume n1+n+1=pq\sqrt{n - 1} + \sqrt{n + 1} = \frac{p}{q}, where p,qp, q are integers, q0q \neq 0, gcd(p,q)=1\gcd(p, q) = 1.

Square both sides:

(n1)+2(n1)(n+1)+(n+1)=p2q2    2n+2n21=p2q2.(n - 1) + 2\sqrt{(n - 1)(n + 1)} + (n + 1) = \frac{p^2}{q^2} \implies 2n + 2\sqrt{n^2 - 1} = \frac{p^2}{q^2}.

Thus:

n21=p22nq22q2.\sqrt{n^2 - 1} = \frac{p^2 - 2n q^2}{2 q^2}.

Suppose n21\sqrt{n^2 - 1} is rational, say ab\frac{a}{b}, gcd(a,b)=1\gcd(a, b) = 1. Then:

n21=a2b2    a2=(n21)b2.n^2 - 1 = \frac{a^2}{b^2} \implies a^2 = (n^2 - 1)b^2.

For n=1n = 1, 0+2=2\sqrt{0} + \sqrt{2} = \sqrt{2}, irrational. For n2n \geq 2, n21=(n1)(n+1)n^2 - 1 = (n - 1)(n + 1) is not a perfect square (since (n1)2<n21<n2(n - 1)^2 < n^2 - 1 < n^2). If a2=(n21)b2a^2 = (n^2 - 1)b^2, n21n^2 - 1 must be a perfect square, a contradiction for n2n \geq 2. Thus, n21\sqrt{n^2 - 1} is irrational, so n1+n+1\sqrt{n - 1} + \sqrt{n + 1} is irrational.

Problem 1.15: Approximation by Rational Numbers

Section titled “Problem 1.15: Approximation by Rational Numbers”

Problem. Given a real xx and an integer N>1N > 1, prove that there exist integers hh and kk with 0<kN0 < k \leq N such that kxh<1/N|kx - h| < 1/N. Hint. Consider the N+1N+1 numbers tx[tx]tx-[tx] for t=0,1,2,,Nt=0,1,2,\dots,N and show that some pair differs by at most 1/N1/N.

Strategy: Use the pigeonhole principle. Consider the fractional parts of 0,x,2x,,Nx0, x, 2x, \ldots, Nx, divide [0,1)[0,1) into NN equal subintervals, and apply the pigeonhole principle to find two numbers in the same subinterval.

Solution: We will use the pigeonhole principle to prove this result.

Consider the N+1N + 1 numbers: 0,x,2x,3x,,Nx0, x, 2x, 3x, \ldots, Nx.

Let’s look at the fractional parts of these numbers. The fractional part of a number yy is yyy - \lfloor y \rfloor, where y\lfloor y \rfloor is the greatest integer less than or equal to yy.

The fractional parts of 0,x,2x,,Nx0, x, 2x, \ldots, Nx all lie in the interval [0,1)[0, 1).

Divide the interval [0,1)[0, 1) into NN equal subintervals:

[0,1/N),[1/N,2/N),,[(N1)/N,1)\begin{aligned} [0, 1/N), [1/N, 2/N), \ldots, [(N-1)/N, 1) \end{aligned}

By the pigeonhole principle, since we have N+1N + 1 numbers and only NN subintervals, at least two of these numbers must fall into the same subinterval.

Let’s say ixix and jxjx (where 0i<jN0 \leq i < j \leq N) have fractional parts in the same subinterval. Then:

(jxjx)(ixix)<1N(ji)x(jxix)<1N\begin{aligned} |(jx - \lfloor jx \rfloor) - (ix - \lfloor ix \rfloor)| &< \frac{1}{N} \\ |(j - i)x - (\lfloor jx \rfloor - \lfloor ix \rfloor)| &< \frac{1}{N} \end{aligned}

Let k=jik = j - i and h=jxixh = \lfloor jx \rfloor - \lfloor ix \rfloor. Then:

kxh<1N\begin{aligned} |kx - h| &< \frac{1}{N} \end{aligned}

Since 0<i<jN0 < i < j \leq N, we have 0<kN0 < k \leq N, and hh is an integer.

Therefore, we have found integers hh and kk with 0<kN0 < k \leq N such that kxh<1/N|kx - h| < 1/N.

Example: If x=πx = \pi and N=5N = 5, we might find that 3π9.42483\pi \approx 9.4248 and 5π15.70805\pi \approx 15.7080 have fractional parts in the same subinterval, giving us 2π6<1/5|2\pi - 6| < 1/5.

Problem 1.16: Infinitely Many Rational Approximations

Section titled “Problem 1.16: Infinitely Many Rational Approximations”

Problem. If xx is irrational, prove that there are infinitely many rational numbers h/kh/k with k>0k > 0 such that xh/k<1/k2|x - h/k| < 1/k^2.

Strategy: We will use the result from Problem 1.15 (Dirichlet’s Approximation Theorem) to construct rational approximations, then use proof by contradiction to show that there must be infinitely many distinct such approximations.

Solution: We will construct an infinite sequence of distinct rational numbers satisfying the condition.

From Problem 1.15 (Dirichlet’s Approximation Theorem), for any integer N>1N > 1, there exist integers hh and kk with 0<kN0 < k \leq N such that kxh<1/N|kx - h| < 1/N. Dividing by kk, we get:

xhk<1Nk\left|x - \frac{h}{k}\right| < \frac{1}{Nk}

Since kNk \leq N, we have 1N1k\frac{1}{N} \leq \frac{1}{k}, which implies 1Nk1k2\frac{1}{Nk} \leq \frac{1}{k^2}. Thus, for any integer N>1N>1, we can find a rational number h/kh/k such that:

xhk<1k2\left|x - \frac{h}{k}\right| < \frac{1}{k^2}

Now we must show that this process can generate infinitely many distinct fractions. Assume, for the sake of contradiction, that there are only a finite number of such rational approximations, say {h1/k1,h2/k2,,hm/km}\{h_1/k_1, h_2/k_2, \ldots, h_m/k_m\}. Since xx is irrational, for any rational number hi/kih_i/k_i, the distance xhi/ki|x - h_i/k_i| is non-zero. Let ϵ\epsilon be the smallest of these non-zero distances:

ϵ=mini=1,,mxhiki>0.\epsilon = \min_{i=1,\dots,m} \left|x - \frac{h_i}{k_i}\right| > 0.

Now, choose an integer NN large enough such that 1/N<ϵ1/N < \epsilon. By the result from Problem 1.15, there exist integers hh' and kk' with 0<kN0 < k' \leq N such that:

kxh<1N|k'x - h'| < \frac{1}{N}

This implies xh/k<1Nk1N|x - h'/k'| < \frac{1}{Nk'} \leq \frac{1}{N}. So we have found a new rational approximation h/kh'/k' such that:

xhk<1N<ϵ\left|x - \frac{h'}{k'}\right| < \frac{1}{N} < \epsilon

Since the approximation error of h/kh'/k' is smaller than ϵ\epsilon, h/kh'/k' cannot be one of the fractions in our finite list {h1/k1,,hm/km}\{h_1/k_1, \ldots, h_m/k_m\}. This contradicts our assumption that we had a complete list of all such approximations. Therefore, there must be infinitely many such rational numbers.

Problem 1.17: Factorial Representation of Rationals (Precise Form)

Section titled “Problem 1.17: Factorial Representation of Rationals (Precise Form)”

Problem. Let xx be a positive rational number of the form

x=k=1nakk!,x = \sum_{k=1}^n \frac{a_k}{k!},

where each aka_k is a nonnegative integer with akk1a_k \leq k - 1 for k2k \geq 2 and an>0a_n > 0. Let [x][x] denote the greatest integer less than or equal to xx. Prove that a1=[x]a_1 = [x], that

ak=[k!x]k[(k1)!x]for k=2,,n,a_k = [k!x] - k[(k - 1)!x] \quad \text{for } k = 2, \dots, n,

and that nn is the smallest integer such that n!xn!x is an integer. Conversely, show that every positive rational number xx can be expressed in this form in one and only one way.

Strategy: For the forward direction, use the properties of the factorial series to show that the fractional part is less than 1, then use the floor function properties. For the converse, use proof by contradiction to show uniqueness by assuming two different representations and finding a contradiction.

Solution: Let x=k=1nakk!x = \sum_{k=1}^n \frac{a_k}{k!} with the given conditions on aka_k.

1. Proof that a1=[x]a_1 = [x]: The sum can be written as x=a1+k=2nakk!x = a_1 + \sum_{k=2}^n \frac{a_k}{k!}. We must show the summation part is a positive fraction less than 1. Since an>0a_n > 0, the sum is positive. We can bound the sum using the property akk1a_k \leq k-1:

k=2nakk!k=2nk1k!<k=2k1k!\sum_{k=2}^n \frac{a_k}{k!} \leq \sum_{k=2}^n \frac{k-1}{k!} < \sum_{k=2}^{\infty} \frac{k-1}{k!}

The infinite sum is a known identity: k=2k1k!=k=2(1(k1)!1k!)\sum_{k=2}^{\infty} \frac{k-1}{k!} = \sum_{k=2}^{\infty} \left(\frac{1}{(k-1)!} - \frac{1}{k!}\right). This is a telescoping series whose sum is the first term, 1/(21)!=11/(2-1)! = 1. Thus, 0<k=2nakk!<10 < \sum_{k=2}^n \frac{a_k}{k!} < 1. This means a1<x<a1+1a_1 < x < a_1 + 1, so by definition, a1=[x]a_1 = [x].

2. Formula for aka_k: Define x1=xa1=k=2nakk!x_1 = x - a_1 = \sum_{k=2}^n \frac{a_k}{k!}. Then k!x1k!x_1 is an integer for knk \ge n. Consider the expression k!xk((k1)!x)=k!(a1+x1)k((k1)!(a1+x1))=ka1k!/k!...k!x - k((k-1)!x) = k!(a_1+x_1) - k((k-1)!(a_1+x_1)) = ka_1k!/k! ... this gets complicated. Let’s use the given formula. Let xk=k!xj=1kajk!j!=j=k+1najk!j!=ak+1k+1+ak+2(k+1)(k+2)+x_k = k!x - \sum_{j=1}^{k} a_j \frac{k!}{j!} = \sum_{j=k+1}^{n} a_j \frac{k!}{j!} = \frac{a_{k+1}}{k+1} + \frac{a_{k+2}}{(k+1)(k+2)} + \dots. From part (1), we know 0xk<10 \le x_k < 1. So [k!x]=j=1kajk!j![k!x] = \sum_{j=1}^{k} a_j \frac{k!}{j!}. Let’s test the formula: ak=[k!x]k[(k1)!x]a_k = [k!x] - k[(k - 1)!x]. We have [k!x]=k!j=1kajj![k!x] = k! \sum_{j=1}^k \frac{a_j}{j!} and [(k1)!x]=(k1)!j=1k1ajj![(k-1)!x] = (k-1)! \sum_{j=1}^{k-1} \frac{a_j}{j!}. So, [k!x]k[(k1)!x]=j=1kajk!j!kj=1k1aj(k1)!j!=(ak+kj=1k1aj(k1)!j!)kj=1k1aj(k1)!j!=ak[k!x] - k[(k-1)!x] = \sum_{j=1}^k a_j \frac{k!}{j!} - k \sum_{j=1}^{k-1} a_j \frac{(k-1)!}{j!} = \left(a_k + k\sum_{j=1}^{k-1} a_j \frac{(k-1)!}{j!}\right) - k\sum_{j=1}^{k-1} a_j \frac{(k-1)!}{j!} = a_k. This proves the formula for aka_k.

3. Minimality of nn: Multiplying xx by n!n! gives n!x=k=1nakn!k!n!x = \sum_{k=1}^n a_k \frac{n!}{k!}. Since knk \le n, each term n!k!\frac{n!}{k!} is an integer, so n!xn!x is an integer. For any m<nm < n, when we compute m!xm!x, the term corresponding to k=nk=n is m!ann!=ann(n1)(m+1)m! \frac{a_n}{n!} = \frac{a_n}{n(n-1)\dots(m+1)}. Since 0<ann10 < a_n \le n-1, this term is a non-integer fraction. Because all other terms for k>mk>m are also fractions and terms for kmk\le m are integers, m!xm!x cannot be an integer. Thus, nn is the smallest such integer.

4. Converse (Uniqueness): Suppose a positive rational number xx has two different representations:

x=k=1nakk!=k=1mbkk!\begin{aligned} x &= \sum_{k=1}^n \frac{a_k}{k!}\\ &= \sum_{k=1}^m \frac{b_k}{k!} \end{aligned}

with the conditions 0akk10 \le a_k \le k-1 for k2k \ge 2, an>0a_n > 0, and similarly for bkb_k. From part (3), nn is the smallest integer such that n!xn!x is an integer, and mm is the smallest integer such that m!xm!x is an integer. This implies n=mn=m.

Let jj be the largest index for which the coefficients differ, so ajbja_j \neq b_j. Assume, without loss of generality, that aj>bja_j > b_j. Since ak=bka_k = b_k for k>jk > j, we can subtract the sums:

k=1jakk!=k=1jbkk!\begin{aligned} \sum_{k=1}^j \frac{a_k}{k!} = \sum_{k=1}^j \frac{b_k}{k!} \end{aligned}

Rearranging the terms, we get:

ajbjj!=k=1j1bkakk!\begin{aligned} \frac{a_j - b_j}{j!} = \sum_{k=1}^{j-1} \frac{b_k - a_k}{k!} \end{aligned}

Multiply both sides by (j1)!(j-1)!:

ajbjj=k=1j1(bkak)(j1)!k!\frac{a_j - b_j}{j} = \sum_{k=1}^{j-1} (b_k - a_k) \frac{(j-1)!}{k!}

The right-hand side is an integer, because for each k{1,,j1}k \in \{1, \dots, j-1\}, k!k! divides (j1)!(j-1)!. Let’s analyze the left-hand side. Since aja_j and bjb_j are integers and aj>bja_j > b_j, we have ajbj1a_j - b_j \geq 1. From the conditions on the coefficients, ajj1a_j \leq j-1 (for j2j \ge 2) and bj0b_j \geq 0. Therefore, 1ajbjj11 \leq a_j - b_j \leq j-1. This implies that for j2j \ge 2, the left-hand side ajbjj\frac{a_j - b_j}{j} is a non-integer fraction, since the numerator is an integer between 11 and j1j-1, and the denominator is jj. This creates a contradiction: the left-hand side cannot be an integer, while the right-hand side must be an integer. For the case j=1j=1, the equation becomes a1b1=0a_1 - b_1 = 0, which contradicts a1b1a_1 \neq b_1. Thus, our assumption that there is a largest index jj where ajbja_j \neq b_j must be false. All coefficients must be identical. The representation is unique.

5. Uniqueness: Suppose xx has two different representations, akk!=bkk!\sum \frac{a_k}{k!} = \sum \frac{b_k}{k!}. Let jj be the largest index where ajbja_j \neq b_j. Assume aj>bja_j > b_j. Then ajbjj!=k=1j1bkakk!\frac{a_j - b_j}{j!} = \sum_{k=1}^{j-1} \frac{b_k - a_k}{k!}. The left side is 1/j!\ge 1/j!. The right side is bounded above by k=1j1k1k!<1/j!\sum_{k=1}^{j-1} \frac{k-1}{k!} < 1/j!, a contradiction. Thus, all coefficients must be the same.

Theorem (Completeness Axiom).

Every nonempty set of real numbers that is bounded above has a supremum.

**Importance:**The Completeness Axiom is the fundamental property that distinguishes the real numbers from the rational numbers. It ensures that the real number system has no “gaps” and is essential for all of calculus and analysis. This axiom is the foundation for proving the existence of limits, continuity, and many other important results in mathematics.

Theorem (Uniqueness of Supremum and Infimum).

If a set has a supremum (infimum), it is unique.

**Importance:**This theorem ensures that suprema and infima are well-defined concepts. Without uniqueness, we couldn’t meaningfully talk about “the” supremum or “the” infimum of a set. This result is essential for the logical consistency of real analysis and is used implicitly in many proofs throughout mathematics.

Theorem (Archimedean Property).

For any positive real numbers aa and bb, there exists a positive integer nn such that na>bna > b.

Theorem (Comparison Property for Suprema).

Let SS and TT be nonempty subsets of R\mathbb{R} such that sts \leq t for all sSs \in S and tTt \in T. If TT has a supremum, then SS has a supremum and supSsupT\sup S \leq \sup T.

**Importance:**This theorem provides a powerful tool for comparing suprema of different sets. It’s essential for many proofs in analysis where we need to establish inequalities between suprema. This result is particularly useful in functional analysis, measure theory, and optimization theory where we often need to compare bounds of different sets or functions.

Problem 1.18: Uniqueness of Supremum and Infimum

Section titled “Problem 1.18: Uniqueness of Supremum and Infimum”

Problem. Show that the sup\sup and inf\inf of a set are uniquely determined whenever they exist.

Strategy: We will prove the uniqueness of supremum by contradiction, assuming there are two different suprema and showing this leads to a contradiction. The same approach applies to infimum.

Solution: We will prove that if a set has a supremum, it is unique. The proof for infimum is similar.

Proof by contradiction: Suppose a set SS has two different suprema, say s1s_1 and s2s_2, with s1<s2s_1 < s_2.

Since s1s_1 is a supremum of SS:

  1. s1s_1 is an upper bound of SS (every element of SS is s1\leq s_1)
  2. s1s_1 is the least upper bound (no number less than s1s_1 is an upper bound)

Since s2s_2 is also a supremum of SS:

  1. s2s_2 is an upper bound of SS (every element of SS is s2\leq s_2)
  2. s2s_2 is the least upper bound (no number less than s2s_2 is an upper bound)

But since s1<s2s_1 < s_2, the number s1s_1 is less than s2s_2 and is also an upper bound of SS. This contradicts the fact that s2s_2 is the least upper bound.

Therefore, our assumption that there are two different suprema is false, and the supremum must be unique.

Alternative proof: Let s1s_1 and s2s_2 both be suprema of SS. Then:

  • s1s_1 is an upper bound, so s2s1s_2 \leq s_1 (since s2s_2 is the least upper bound)
  • s2s_2 is an upper bound, so s1s2s_1 \leq s_2 (since s1s_1 is the least upper bound)

Therefore, s1=s2s_1 = s_2.

For infimum: The same argument applies to infimum. If a set has two infima i1i_1 and i2i_2, then:

  • i1i_1 is a lower bound, so i1i2i_1 \leq i_2 (since i1i_1 is the greatest lower bound)
  • i2i_2 is a lower bound, so i2i1i_2 \leq i_1 (since i2i_2 is the greatest lower bound)

Therefore, i1=i2i_1 = i_2.

Problem 1.19: Finding Supremum and Infimum

Section titled “Problem 1.19: Finding Supremum and Infimum”

Problem. Find the sup\sup and inf\inf of each of the following sets:

  1. All numbers of the form 2p+3q+5r2^{-p} + 3^{-q} + 5^{-r} for positive integers p,q,rp, q, r.
  2. S={x:3x210x+3<0}S = \{x : 3x^2 - 10x + 3 < 0\}.
  3. S={x:(xa)(xb)(xc)(xd)<0}S = \{x : (x - a)(x - b)(x - c)(x - d) < 0\} where a<b<c<da < b < c < d.

Strategy: For (a), analyze the range of each exponential term and find the maximum and minimum values. For (b), solve the quadratic inequality to find the interval. For (c), use the sign changes of the polynomial at its roots to determine the intervals where the product is negative.

Solution:

**1. Numbers of the form 2^{-p** + 3^{-q} + 5^{-r}:}

Let’s analyze the range of each term:

  • 2p2^{-p} ranges from 12\frac{1}{2} (when p=1p = 1) to 00 (as pp \to \infty)
  • 3q3^{-q} ranges from 13\frac{1}{3} (when q=1q = 1) to 00 (as qq \to \infty)
  • 5r5^{-r} ranges from 15\frac{1}{5} (when r=1r = 1) to 00 (as rr \to \infty)

Therefore:

  • The maximum value occurs when p=q=r=1p = q = r = 1: 12+13+15=15+10+630=3130\frac{1}{2} + \frac{1}{3} + \frac{1}{5} = \frac{15 + 10 + 6}{30} = \frac{31}{30}
  • The minimum value occurs as p,q,rp, q, r \to \infty: 0+0+0=00 + 0 + 0 = 0

So sup=\sup = **\frac{31**{30}} and inf=\inf = **0**.

**2. Set S={x:3x210x+3<0\*S = \{x : 3x^2 - 10x + 3 < 0\**:}

First, let’s find the roots of 3x210x+3=03x^2 - 10x + 3 = 0:

x=10±100366=10±646=10±86=186=3or26=13\begin{aligned} x &= \frac{10 \pm \sqrt{100 - 36}}{6} \\ &= \frac{10 \pm \sqrt{64}}{6} \\ &= \frac{10 \pm 8}{6} \\ &= \frac{18}{6} = 3 \quad \text{or} \quad \frac{2}{6} = \frac{1}{3} \end{aligned}

Since the coefficient of x2x^2 is positive, the parabola opens upward. The inequality 3x210x+3<03x^2 - 10x + 3 < 0 holds between the roots.

Therefore, S=(13,3)S = (\frac{1}{3}, 3), so sup=\sup = **3** and inf=\inf = **\frac{1**{3}}.

Visualization for part (b):

[TikZ diagram omitted in web version]

This visualization shows the quadratic function y=3x210x+3y = 3x^2 - 10x + 3 and highlights the interval where the inequality 3x210x+3<03x^2 - 10x + 3 < 0 holds, which is between the roots 13\frac{1}{3} and 33.

**3. Set S={x:(xa)(xb)(xc)(xd)<0\*S = \{x : (x - a)(x - b)(x - c)(x - d) < 0\** where a<b<c<da < b < c < d:}

The expression (xa)(xb)(xc)(xd)(x - a)(x - b)(x - c)(x - d) changes sign at each root a,b,c,da, b, c, d.

Starting from -\infty:

  • For x<ax < a: all factors are negative, so the product is positive
  • For a<x<ba < x < b: one factor is positive, three negative, so product is negative
  • For b<x<cb < x < c: two factors positive, two negative, so product is positive
  • For c<x<dc < x < d: three factors positive, one negative, so product is negative
  • For x>dx > d: all factors are positive, so product is positive

Therefore, S=(a,b)(c,d)S = (a, b) \cup (c, d).

So sup=\sup = **d** and inf=\inf = **a**.

Visualization for part (c):

[TikZ diagram omitted in web version]

This visualization shows the fourth-degree polynomial (xa)(xb)(xc)(xd)(x-a)(x-b)(x-c)(x-d) and highlights the intervals where the inequality (xa)(xb)(xc)(xd)<0(x-a)(x-b)(x-c)(x-d) < 0 holds. The polynomial changes sign at each root, creating alternating positive and negative intervals.

Problem 1.20: Comparison Property for Suprema

Section titled “Problem 1.20: Comparison Property for Suprema”

Problem. Let ( S ) and ( T ) be nonempty subsets of ( \mathbb{R} ) such that ( s \leq t ) for all ( s \in S ) and ( t \in T ). Suppose ( T ) has a supremum. Then ( S ) has a supremum and

supSsupT.\sup S \leq \sup T.

Strategy: We will first show that SS has a supremum by using the completeness axiom, then prove that supSsupT\sup S \leq \sup T by showing that supS\sup S is a lower bound for TT and using the definition of supremum.

Solution: Let SS and TT be nonempty subsets of R\mathbb{R} with the property that for every sSs \in S and tTt \in T, we have sts \leq t.

  1. Existence of supS\sup S: Since TT is nonempty, we can pick an arbitrary element t0Tt_0 \in T. By the given property, for every sSs \in S, we have st0s \leq t_0. This shows that SS is bounded above (by any element of TT). Since SS is also nonempty and bounded above, the completeness axiom of R\mathbb{R} guarantees that supS\sup S exists. Let’s call it α=supS\alpha = \sup S.

  2. Proof that supSsupT\sup S \leq \sup T: Let α=supS\alpha = \sup S and β=supT\beta = \sup T. From step 1, we know that any element tTt \in T is an upper bound for the set SS. Since α\alpha is the least upper bound of SS, it must be less than or equal to any other upper bound of SS. Therefore, for any tTt \in T, we must have:

αt\alpha \leq t

This inequality shows that α\alpha is a lower bound for the set TT. Now, by definition, β=supT\beta = \sup T is the least upper bound of TT. As an upper bound for TT, β\beta must be greater than or equal to every element of TT. More importantly, it must be greater than or equal to any lower bound of TT. Since we have established that α\alpha is a lower bound for TT, it must follow that:

αβ\alpha \leq \beta

Substituting the definitions of α\alpha and β\beta, we get:

supSsupT\sup S \leq \sup T

This completes the proof.

Problem. Let ( A ) and ( B ) be two sets of positive real numbers, each bounded above. Let ( a = \sup A ), ( b = \sup B ). Define

C={xy:xA,yB}.C = \{ xy : x \in A,\, y \in B \}.

Prove that

supC=ab.\sup C = ab.

Strategy: We will show that abab is an upper bound for CC, then prove it is the least upper bound by using the definition of supremum and constructing elements of CC that are arbitrarily close to abab.

Solution:

Since ( A ) and ( B ) are sets of positive real numbers bounded above, their suprema ( a = \sup A ) and ( b = \sup B ) exist and are finite.

We are to prove that:

supC=ab.\sup C = ab.

Step 1: Show that ( ab ) is an upper bound for ( C ).

Let ( x \in A ), ( y \in B ). Since ( x \leq a ) and ( y \leq b ), we have:

xyab.xy \leq ab.

Therefore, every element ( c \in C ) satisfies ( c \leq ab ), so ( ab ) is an upper bound for ( C ).

Step 2: Show that ( ab ) is the least upper bound.

Let ( \varepsilon > 0 ). Since ( a = \sup A ), there exists ( x_\varepsilon \in A ) such that:

xε>aε2b.x_\varepsilon > a - \frac{\varepsilon}{2b}.

Similarly, since ( b = \sup B ), there exists ( y_\varepsilon \in B ) such that:

yε>bε2a.y_\varepsilon > b - \frac{\varepsilon}{2a}.

Now consider:

xεyε>(aε2b)(bε2a)=abε2ε2+ε24ab.x_\varepsilon y_\varepsilon > \left(a - \frac{\varepsilon}{2b}\right)\left(b - \frac{\varepsilon}{2a}\right) = ab - \frac{\varepsilon}{2} - \frac{\varepsilon}{2} + \frac{\varepsilon^2}{4ab}.

Since ( \frac{\varepsilon^2}{4ab} > 0 ), we have:

xεyε>abε.x_\varepsilon y_\varepsilon > ab - \varepsilon.

Therefore, for every ( \varepsilon > 0 ), there exists ( c \in C ) such that ( c > ab - \varepsilon ). Hence, ( ab ) is the least upper bound of ( C ).

supC=ab\boxed{\sup C = ab}

Problem 1.22: Representation of Rationals in Base ( k )

Section titled “Problem 1.22: Representation of Rationals in Base ( k )”

Problem. Given ( x \geq 0 ) and an integer ( k \geq 2 ), let ( a_0 ) denote the largest integer ( \leq x ), and, assuming that ( a_0, a_1, \dots, a_{n-1} ) have been defined, let ( a_n ) denote the largest integer such that

a0+a1k+a2k2++anknx.a_0 + \frac{a_1}{k} + \frac{a_2}{k^2} + \cdots + \frac{a_n}{k^n} \leq x.
  1. [(a)] Prove that ( 0 \leq a_i \leq k - 1 ) for each ( i = 1, 2, \dots ).
  2. [(b)] Let ( r_n = a_0 + a_1 k^{-1} + a_2 k^{-2} + \cdots + a_n k^{-n} ) and show that ( x = \sup { r_n } ), the supremum of the set of rational numbers ( r_1, r_2, \dots ).

Strategy: For (a), use the definition of ana_n as the largest integer satisfying the condition and show that choosing an+1a_n + 1 would violate it. For (b), use the Archimedean property and proof by contradiction to show that the supremum equals xx.

Solution: Let ( r_n = \sum_{i=0}^n \frac{a_i}{k^i} ). By definition, ( a_n ) is the largest integer such that ( r_n \leq x ).

(a) Show ( 0 \leq a_i \leq k - 1 ): Since ( a_n ) is the largest integer satisfying the condition, choosing ( a_n + 1 ) would violate it:

rn1+an+1kn>xr_{n-1} + \frac{a_n + 1}{k^n} > x

From the definition of (a_{n-1}), we know it was the largest integer such that ( r_{n-1} \le x ). This implies xrn1<1kn1x - r_{n-1} < \frac{1}{k^{n-1}}. Now, from the definition of ana_n, we have rn1+anknxr_{n-1} + \frac{a_n}{k^n} \leq x, which implies ankn(xrn1)a_n \leq k^n(x - r_{n-1}). Combining these facts:

ankn(xrn1)<kn(1kn1)=k.a_n \leq k^n(x - r_{n-1}) < k^n\left(\frac{1}{k^{n-1}}\right) = k.

Since ana_n is an integer and an<ka_n < k, we must have ank1a_n \leq k-1. Also, ana_n must be non-negative, otherwise we could choose an=0a_n=0 to get a larger (or equal) sum rnr_n that is still less than or equal to xx, contradicting the “largest integer” definition if the original ana_n were negative. Thus, 0ank10 \leq a_n \leq k-1.

*(b) Show that ( x = \sup { r_n * ):} The sequence {rn}\{r_n\} is non-decreasing by construction, since an0a_n \ge 0. It is also bounded above by xx. Therefore, its supremum exists; let r=sup{rn}r = \sup\{r_n\}. We know rxr \le x. We will prove r=xr=x by contradiction. Assume r<xr < x. Let δ=xr>0\delta = x - r > 0. By the Archimedean property, we can choose an integer NN large enough such that 1kN<δ\frac{1}{k^N} < \delta. From the definition of aNa_N, we know rN=rN1+aNkNxr_N = r_{N-1} + \frac{a_N}{k^N} \le x and rN1+aN+1kN>xr_{N-1} + \frac{a_N+1}{k^N} > x. The second inequality rearranges to xrN<1kNx - r_N < \frac{1}{k^N}. Since r=sup{rn}r = \sup\{r_n\}, we know rNrr_N \leq r. Therefore, xrxrN<1kNx - r \leq x - r_N < \frac{1}{k^N}. Substituting δ=xr\delta = x-r, we get δ<1kN\delta < \frac{1}{k^N}. But we chose NN such that 1kN<δ\frac{1}{k^N} < \delta. This gives δ<1kN<δ\delta < \frac{1}{k^N} < \delta, a contradiction. Thus, our assumption must be false, and x=r=sup{rn}x = r = \sup\{r_n\}.

Additional Theorems for Inequalities and Identities

Section titled “Additional Theorems for Inequalities and Identities”

Theorem (Binomial Theorem).

For any real numbers aa and bb and positive integer nn:

(a+b)n=k=0n(nk)ankbk(a + b)^n = \sum_{k=0}^n \binom{n}{k} a^{n-k} b^k

**Importance:**The Binomial Theorem is one of the most fundamental results in algebra and combinatorics. It provides a formula for expanding powers of binomials and is essential for understanding polynomials, probability theory, and many areas of mathematics. The binomial coefficients that appear in this theorem are fundamental in combinatorics and have applications throughout mathematics.

Theorem (Pigeonhole Principle).

If nn objects are placed into mm containers where n>mn > m, then at least one container must contain more than one object.

**Importance:**The Pigeonhole Principle is a simple but powerful tool in combinatorics and discrete mathematics. It provides a way to prove the existence of certain patterns or properties without explicitly constructing them. This principle is essential for many existence proofs and has applications in computer science, number theory, and many other areas.

Theorem (Dirichlet’s Approximation Theorem).

For any real number xx and positive integer NN, there exist integers hh and kk with 0<kN0 < k \leq N such that kxh<1/N|kx - h| < 1/N.

**Importance:**Dirichlet’s Approximation Theorem is a fundamental result in Diophantine approximation that shows how well real numbers can be approximated by rational numbers. It’s essential for understanding the distribution of rational numbers and has applications in number theory, analysis, and many areas of mathematics. This theorem is the foundation for more advanced results in Diophantine approximation.

Problem. Prove Lagrange’s identity for real numbers:

(k=1nakbk)2=(k=1nak2)(k=1nbk2)1k<jn(akbjajbk)2.\left( \sum_{k=1}^n a_k b_k \right)^2 = \left( \sum_{k=1}^n a_k^2 \right)\left( \sum_{k=1}^n b_k^2 \right) - \sum_{1 \leq k < j \leq n} (a_k b_j - a_j b_k)^2.

Strategy: We will prove this identity by expanding both sides and showing they are equal. We’ll expand the left-hand side as a double sum and the right-hand side by expanding the product and the squared terms, then show that all terms cancel appropriately.

Solution: We will prove Lagrange’s identity by expanding both sides and showing they are equal.

Let’s start by expanding the left-hand side:

(k=1nakbk)2=(k=1nakbk)(j=1najbj)=k=1nj=1nakbkajbj=k=1nak2bk2+21k<jnakbkajbj\begin{aligned} \left( \sum_{k=1}^n a_k b_k \right)^2 &= \left( \sum_{k=1}^n a_k b_k \right) \left( \sum_{j=1}^n a_j b_j \right) \\ &= \sum_{k=1}^n \sum_{j=1}^n a_k b_k a_j b_j \\ &= \sum_{k=1}^n a_k^2 b_k^2 + 2 \sum_{1 \leq k < j \leq n} a_k b_k a_j b_j \end{aligned}

Now let’s expand the right-hand side:

(k=1nak2)(k=1nbk2)1k<jn(akbjajbk)2=(k=1nak2)(j=1nbj2)1k<jn(ak2bj22akbjajbk+aj2bk2)=k=1nj=1nak2bj21k<jnak2bj2+21k<jnakbjajbk1k<jnaj2bk2\begin{aligned} &\left( \sum_{k=1}^n a_k^2 \right)\left( \sum_{k=1}^n b_k^2 \right) - \sum_{1 \leq k < j \leq n} (a_k b_j - a_j b_k)^2 \\ &= \left( \sum_{k=1}^n a_k^2 \right)\left( \sum_{j=1}^n b_j^2 \right) - \sum_{1 \leq k < j \leq n} (a_k^2 b_j^2 - 2a_k b_j a_j b_k + a_j^2 b_k^2) \\ &= \sum_{k=1}^n \sum_{j=1}^n a_k^2 b_j^2 - \sum_{1 \leq k < j \leq n} a_k^2 b_j^2 + 2 \sum_{1 \leq k < j \leq n} a_k b_j a_j b_k - \sum_{1 \leq k < j \leq n} a_j^2 b_k^2 \end{aligned}

Let’s simplify this step by step. First, note that:

k=1nj=1nak2bj2=k=1nak2bk2+1k<jnak2bj2+1k<jnaj2bk2=k=1nak2bk2+1k<jn(ak2bj2+aj2bk2)\begin{aligned} \sum_{k=1}^n \sum_{j=1}^n a_k^2 b_j^2 &= \sum_{k=1}^n a_k^2 b_k^2 + \sum_{1 \leq k < j \leq n} a_k^2 b_j^2 + \sum_{1 \leq k < j \leq n} a_j^2 b_k^2 \\ &= \sum_{k=1}^n a_k^2 b_k^2 + \sum_{1 \leq k < j \leq n} (a_k^2 b_j^2 + a_j^2 b_k^2) \end{aligned}

Substituting this back into our expression:

k=1nak2bk2+1k<jn(ak2bj2+aj2bk2)1k<jnak2bj2+21k<jnakbjajbk1k<jnaj2bk2=k=1nak2bk2+21k<jnakbjajbk\begin{aligned} &\sum_{k=1}^n a_k^2 b_k^2 + \sum_{1 \leq k < j \leq n} (a_k^2 b_j^2 + a_j^2 b_k^2) - \sum_{1 \leq k < j \leq n} a_k^2 b_j^2 + 2 \sum_{1 \leq k < j \leq n} a_k b_j a_j b_k \\ &- \sum_{1 \leq k < j \leq n} a_j^2 b_k^2 \\ =& \sum_{k=1}^n a_k^2 b_k^2 + 2 \sum_{1 \leq k < j \leq n} a_k b_j a_j b_k \end{aligned}

This is exactly the same as our expanded left-hand side! Therefore, Lagrange’s identity is proven.

Alternative Proof using Determinants: We can also prove this using the fact that the determinant of a 2×2 matrix is zero if and only if its rows are linearly dependent.

Consider the matrix:

(akbkajbj)\begin{pmatrix} a_k & b_k \\ a_j & b_j \end{pmatrix}

The determinant of this matrix is akbjajbka_k b_j - a_j b_k. If we square this determinant and sum over all pairs (k,j)(k,j) with k<jk < j, we get the right-hand side of Lagrange’s identity.

The left-hand side represents the square of the dot product of the vectors a=(a1,a2,,an)\mathbf{a} = (a_1, a_2, \ldots, a_n) and b=(b1,b2,,bn)\mathbf{b} = (b_1, b_2, \ldots, b_n).

The identity shows that the square of the dot product equals the product of the squared magnitudes minus the sum of squared determinants of all 2×2 submatrices formed by pairs of components.

Problem. Prove that for arbitrary real numbers ( a_k, b_k, c_k ) we have

(k=1nakbkck)4(k=1nak4)(k=1nbk2)2(k=1nck4).\left( \sum_{k=1}^n a_k b_k c_k \right)^4 \leq \left( \sum_{k=1}^n a_k^4 \right) \left( \sum_{k=1}^n b_k^2 \right)^2 \left( \sum_{k=1}^n c_k^4 \right).

Strategy: We will apply the Cauchy-Schwarz inequality twice. First, we’ll group (akck)(a_k c_k) and bkb_k, then apply Cauchy-Schwarz to the resulting term k=1nak2ck2\sum_{k=1}^n a_k^2 c_k^2 by treating it as the dot product of sequences {ak2}\{a_k^2\} and {ck2}\{c_k^2\}.

Solution: We will prove this inequality by applying the Cauchy-Schwarz inequality twice. First, group the terms as (akck)(a_k c_k) and bkb_k. Applying the Cauchy-Schwarz inequality to the sequences {akck}\{a_k c_k\} and {bk}\{b_k\} gives:

(k=1n(akck)bk)2(k=1n(akck)2)(k=1nbk2)=(k=1nak2ck2)(k=1nbk2).\left( \sum_{k=1}^n (a_k c_k) b_k \right)^2 \leq \left( \sum_{k=1}^n (a_k c_k)^2 \right) \left( \sum_{k=1}^n b_k^2 \right) = \left( \sum_{k=1}^n a_k^2 c_k^2 \right) \left( \sum_{k=1}^n b_k^2 \right).

Next, we apply the Cauchy-Schwarz inequality to the term k=1nak2ck2\sum_{k=1}^n a_k^2 c_k^2, treating it as the dot product of sequences {ak2}\{a_k^2\} and {ck2}\{c_k^2\}:

(k=1nak2ck2)2(k=1n(ak2)2)(k=1n(ck2)2)=(k=1nak4)(k=1nck4).\left( \sum_{k=1}^n a_k^2 c_k^2 \right)^2 \leq \left( \sum_{k=1}^n (a_k^2)^2 \right) \left( \sum_{k=1}^n (c_k^2)^2 \right) = \left( \sum_{k=1}^n a_k^4 \right) \left( \sum_{k=1}^n c_k^4 \right).

This implies:

k=1nak2ck2(k=1nak4)1/2(k=1nck4)1/2.\sum_{k=1}^n a_k^2 c_k^2 \leq \left( \sum_{k=1}^n a_k^4 \right)^{1/2} \left( \sum_{k=1}^n c_k^4 \right)^{1/2}.

Now, substitute this result back into our first inequality:

(k=1nakbkck)2(k=1nak4)1/2(k=1nck4)1/2(k=1nbk2).\left( \sum_{k=1}^n a_k b_k c_k \right)^2 \leq \left( \sum_{k=1}^n a_k^4 \right)^{1/2} \left( \sum_{k=1}^n c_k^4 \right)^{1/2} \left( \sum_{k=1}^n b_k^2 \right).

Finally, squaring both sides gives the desired result:

(k=1nakbkck)4(k=1nak4)(k=1nck4)(k=1nbk2)2.\left( \sum_{k=1}^n a_k b_k c_k \right)^4 \leq \left( \sum_{k=1}^n a_k^4 \right) \left( \sum_{k=1}^n c_k^4 \right) \left( \sum_{k=1}^n b_k^2 \right)^2.

Problem. Prove Minkowski’s inequality:

(k=1n(ak+bk)2)1/2(k=1nak2)1/2+(k=1nbk2)1/2.\left( \sum_{k=1}^n (a_k + b_k)^2 \right)^{1/2} \leq \left( \sum_{k=1}^n a_k^2 \right)^{1/2} + \left( \sum_{k=1}^n b_k^2 \right)^{1/2}.

Strategy: We will expand the left-hand side, apply the Cauchy-Schwarz inequality to the cross term, then complete the square to obtain the desired inequality.

Solution:

Let ( A = \left( \sum a_k^2 \right)^{1/2} ), ( B = \left( \sum b_k^2 \right)^{1/2} ), and expand the square:

(ak+bk)2=ak2+2akbk+bk2=A2+2akbk+B2.\sum (a_k + b_k)^2 = \sum a_k^2 + 2\sum a_k b_k + \sum b_k^2 = A^2 + 2\sum a_k b_k + B^2.

Apply Cauchy–Schwarz:

akbkAB.\sum a_k b_k \leq A B.

Thus,

(ak+bk)2A2+2AB+B2=(A+B)2.\sum (a_k + b_k)^2 \leq A^2 + 2AB + B^2 = (A + B)^2.

Taking square roots:

((ak+bk)2)1/2A+B.\left( \sum (a_k + b_k)^2 \right)^{1/2} \leq A + B.

Problem 1.26: Chebyshev’s Sum Inequality

Section titled “Problem 1.26: Chebyshev’s Sum Inequality”

Problem. If ( a_1 \geq a_2 \geq \cdots \geq a_n ) and ( b_1 \geq b_2 \geq \cdots \geq b_n ), prove that

(k=1nak)(k=1nbk)nk=1nakbk.\left( \sum_{k=1}^n a_k \right)\left( \sum_{k=1}^n b_k \right) \leq n \sum_{k=1}^n a_k b_k.

Strategy: We will consider the double summation S=i=1nj=1n(aiaj)(bibj)S = \sum_{i=1}^n \sum_{j=1}^n (a_i - a_j)(b_i - b_j) and show that it is non-negative due to the ordering of the sequences, then expand it to obtain the desired inequality.

Solution: Consider the double summation

S=i=1nj=1n(aiaj)(bibj).S = \sum_{i=1}^n \sum_{j=1}^n (a_i - a_j)(b_i - b_j).

Since the sequences {ak}\{a_k\} and {bk}\{b_k\} are sorted in the same order (both non-increasing), the terms (aiaj)(a_i - a_j) and (bibj)(b_i - b_j) always have the same sign. If i>ji>j, then aiaja_i \le a_j and bibjb_i \le b_j, so both differences are non-positive. If i<ji<j, both are non-negative. Therefore, their product is always non-negative:

(aiaj)(bibj)0.(a_i - a_j)(b_i - b_j) \geq 0.

This implies that the total sum SS must be non-negative, S0S \geq 0.

Now, let’s expand the sum:

S=i=1nj=1n(aibiaibjajbi+ajbj)=i=1nj=1naibii=1nj=1naibji=1nj=1najbi+i=1nj=1najbj\begin{aligned} S &= \sum_{i=1}^n \sum_{j=1}^n (a_i b_i - a_i b_j - a_j b_i + a_j b_j) \\ &= \sum_{i=1}^n \sum_{j=1}^n a_i b_i - \sum_{i=1}^n \sum_{j=1}^n a_i b_j - \sum_{i=1}^n \sum_{j=1}^n a_j b_i + \sum_{i=1}^n \sum_{j=1}^n a_j b_j \end{aligned}

We evaluate each double summation:

  • ( \sum_{i=1}^n \sum_{j=1}^n a_i b_i = \sum_{i=1}^n \left( n \cdot a_i b_i \right) = n \sum_{i=1}^n a_i b_i )
  • ( \sum_{i=1}^n \sum_{j=1}^n a_i b_j = \left( \sum_{i=1}^n a_i \right) \left( \sum_{j=1}^n b_j \right) )
  • ( \sum_{i=1}^n \sum_{j=1}^n a_j b_i = \left( \sum_{j=1}^n a_j \right) \left( \sum_{i=1}^n b_i \right) )
  • ( \sum_{i=1}^n \sum_{j=1}^n a_j b_j = \sum_{j=1}^n \left( n \cdot a_j b_j \right) = n \sum_{j=1}^n a_j b_j )

Substituting these back into the expression for SS:

S=nakbk(ak)(bk)(ak)(bk)+nakbkS = n \sum a_k b_k - \left(\sum a_k\right)\left(\sum b_k\right) - \left(\sum a_k\right)\left(\sum b_k\right) + n \sum a_k b_k S=2nk=1nakbk2(k=1nak)(k=1nbk)S = 2n \sum_{k=1}^n a_k b_k - 2 \left( \sum_{k=1}^n a_k \right) \left( \sum_{k=1}^n b_k \right)

Since we established that S0S \geq 0:

2nk=1nakbk2(k=1nak)(k=1nbk)02n \sum_{k=1}^n a_k b_k - 2 \left( \sum_{k=1}^n a_k \right) \left( \sum_{k=1}^n b_k \right) \geq 0

Dividing by 2 and rearranging gives the desired inequality:

nk=1nakbk(k=1nak)(k=1nbk).n \sum_{k=1}^n a_k b_k \geq \left( \sum_{k=1}^n a_k \right) \left( \sum_{k=1}^n b_k \right).

Problem 1.27: Express Complex Numbers in a+bia + bi Form

Section titled “Problem 1.27: Express Complex Numbers in a+bia + bia+bi Form”

Problem. Express the following complex numbers in the form ( a + bi ):

  1. [(a)] ( (1 + i)^3 )
  2. [(b)] ( \frac{2 + 3i}{3 - 4i} )
  3. [(c)] ( i^5 + i^{16} )
  4. [(d)] ( \frac{1}{2}(1 + i)(1 + i^{-8}) )

Strategy: We will use the properties of complex numbers, including i2=1i^2 = -1, i4=1i^4 = 1, and the fact that powers of ii cycle every 4. For division, we’ll rationalize the denominator by multiplying by the complex conjugate.

Solution:

  • [(a)] ( (1 + i)^3 = (1 + i)^2 (1 + i) = (2i)(1 + i) = 2i + 2i^2 = 2i - 2 = -2 + 2i )
  • [(b)] Rationalize the denominator:
2+3i34i3+4i3+4i=(2+3i)(3+4i)9+16=6+8i+9i+12i225=6+17i25=625+1725i\begin{aligned} \frac{2 + 3i}{3 - 4i} \cdot \frac{3 + 4i}{3 + 4i} =& \frac{(2 + 3i)(3 + 4i)}{9 + 16} = \frac{6 + 8i + 9i + 12i^2}{25} \\ =& \frac{-6 + 17i}{25} = -\frac{6}{25} + \frac{17}{25}i \end{aligned}
  • [(c)] ( i^5 = i ), since ( i^4 = 1 ), and ( i^{16} = (i^4)^4 = 1 ), so:
i5+i16=i+1=1+ii^5 + i^{16} = i + 1 = 1 + i
  • [(d)] ( \frac{1}{2}(1 + i)(1 + i^{-8}) ), note that ( i^{-8} = (i^4)^{-2} = 1^{-2} = 1 ), so:
12(1+i)(1+1)=12(1+i)(2)=12(2+2i)=1+i\frac{1}{2}(1 + i)(1 + 1) = \frac{1}{2}(1 + i)(2) = \frac{1}{2}(2 + 2i) = 1 + i

Problem. In each case, determine all real ( x ) and ( y ) which satisfy the given relation:

  1. [(a)] ( x + iy = |x - iy| )
  2. [(b)] ( x + iy = (x - iy)^2 )
  3. [(c)] ( \sum_{k=0}^{100} i^k = x + iy )

Strategy: For each equation, we’ll equate the real and imaginary parts. For (a), we’ll use the fact that the right-hand side is real and nonnegative. For (b), we’ll expand the square and solve the resulting system. For (c), we’ll use the cyclic nature of powers of ii.

Solution:

  • [(a)] RHS is real and nonnegative. LHS is complex. For equality, imaginary part must vanish:
Im(x+iy)=y=0,and x=xx0.\text{Im}(x + iy) = y = 0, \quad \text{and } x = |x| \Rightarrow x \geq 0.

So solution: ( y = 0,, x \geq 0 )

  • [(b)] Compute RHS:
(xiy)2=x22ixyy2=(x2y2)2ixy.(x - iy)^2 = x^2 - 2ixy - y^2 = (x^2 - y^2) - 2ixy.

Set equal to ( x + iy ), equate real and imaginary parts:

x=x2y2,y=2xy.x = x^2 - y^2,\quad y = -2xy.

From second equation: ( y = -2xy \Rightarrow y(1 + 2x) = 0 \Rightarrow y = 0 ) or ( x = -\frac{1}{2} )

If ( y = 0 ), then first equation: ( x = x^2 \Rightarrow x(x - 1) = 0 \Rightarrow x = 0 ) or ( x = 1 )

If ( x = -\frac{1}{2} ), then first equation:

x=x2y212=14y2y2=34y=±32x = x^2 - y^2 \Rightarrow -\frac{1}{2} = \frac{1}{4} - y^2 \Rightarrow y^2 = \frac{3}{4} \Rightarrow y = \pm \frac{\sqrt{3}}{2}

So all solutions:

(x,y)=(0,0),(1,0),(12,±32)(x,y) = (0,0), (1,0), \left(-\frac{1}{2}, \pm \frac{\sqrt{3}}{2} \right)
  • [(c)] The powers of ( i ) cycle every 4: ( i^0 = 1, i^1 = i, i^2 = -1, i^3 = -i )

There are ( 101 ) terms, which form 25 full cycles and one leftover term ( i^{100} \equiv i^0 = 1 )

Each full cycle sums to 0. So total sum:

k=0100ik=250+1=1x=1,y=0.\sum_{k=0}^{100} i^k = 25 \cdot 0 + 1 = 1 \Rightarrow x = 1, y = 0.

Problem 1.29: Basic Identities for Complex Conjugates

Section titled “Problem 1.29: Basic Identities for Complex Conjugates”

Problem. If ( z = x + iy ), where ( x ) and ( y ) are real, the complex conjugate of ( z ) is ( \overline{z} = x - iy ). Prove the following:

  1. ( \overline{z_1 + z_2} = \overline{z_1} + \overline{z_2} ),
  2. ( \overline{z_1 z_2} = \overline{z_1} \cdot \overline{z_2} ),
  3. ( z \cdot \overline{z} = |z|^2 ),
  4. ( z + \overline{z} ) is twice the real part of ( z ),
  5. ( \frac{z - \overline{z}}{i} ) is twice the imaginary part of ( z ).

Strategy: We will prove each identity by using the definition of complex conjugate and performing the necessary algebraic manipulations. For each part, we’ll work with the real and imaginary components explicitly.

Solution: Let ( z = x + iy ) and ( w = u + iv ) be two complex numbers.

  1. Conjugate of a sum:
z1+z2=(x1+x2)+i(y1+y2)=(x1+x2)i(y1+y2)=z1+z2.\overline{z_1 + z_2} = \overline{(x_1 + x_2) + i(y_1 + y_2)} = (x_1 + x_2) - i(y_1 + y_2) = \overline{z_1} + \overline{z_2}.
  1. Conjugate of a product:
z1z2=(x1+iy1)(x2+iy2)=(x1x2y1y2+i(x1y2+x2y1))=x1x2y1y2i(x1y2+x2y1)=z1z2.\begin{aligned} \overline{z_1 z_2} =& \overline{(x_1 + iy_1)(x_2 + iy_2)} = \overline{(x_1 x_2 - y_1 y_2 + i(x_1 y_2 + x_2 y_1))} \\ =& x_1 x_2 - y_1 y_2 - i(x_1 y_2 + x_2 y_1) = \overline{z_1} \cdot \overline{z_2}. \end{aligned}
  1. Modulus squared:
zz=(x+iy)(xiy)=x2+y2=z2.z \cdot \overline{z} = (x + iy)(x - iy) = x^2 + y^2 = |z|^2.
  1. Twice the real part:
z+z=(x+iy)+(xiy)=2x=2(z).z + \overline{z} = (x + iy) + (x - iy) = 2x = 2 \Re(z).
  1. Twice the imaginary part:
zzi=(x+iy)(xiy)i=2iyi=2y=2(z).\frac{z - \overline{z}}{i} = \frac{(x + iy) - (x - iy)}{i} = \frac{2iy}{i} = 2y = 2 \Im(z).

Problem 1.30: Geometric Descriptions of Complex Sets

Section titled “Problem 1.30: Geometric Descriptions of Complex Sets”

Problem. Describe geometrically the set of complex numbers ( z ) which satisfies each of the following conditions:

  1. ( |z| = 1 ),
  2. ( |z| < 1 ),
  3. ( |z| \leq 1 ),
  4. ( z + \overline{z} = 1 ),
  5. ( z - \overline{z} = i ),
  6. ( \overline{z} + z = |z|^2 ).

Strategy: We will use the properties of complex conjugates and the relationship between complex numbers and their real/imaginary parts to translate each condition into geometric terms. For the last condition, we’ll complete the square to identify the geometric shape.

Solution:

  1. The unit circle centered at the origin.
  2. The open unit disk centered at the origin.
  3. The closed unit disk centered at the origin.
  4. ( 2 \Re(z) = 1 \Rightarrow \Re(z) = \frac{1}{2} ): a vertical line in the complex plane.
  5. ( 2i \Im(z) = i \Rightarrow \Im(z) = \frac{1}{2} ): a horizontal line.
  6. Let ( z = x + iy ), where ( x, y \in \mathbb{R} ). Then:
z+z=(x+iy)+(xiy)=2x,z2=x2+y2.\begin{aligned} z + \overline{z} &= (x + iy) + (x - iy) = 2x, \\ |z|^2 &= x^2 + y^2. \end{aligned}

So the equation becomes:

2x=x2+y2.2x = x^2 + y^2.

Rewriting this:

x22x+y2=0.x^2 - 2x + y^2 = 0.

We now complete the square on the ( x )-terms:

x22x=(x1)21,x^2 - 2x = (x - 1)^2 - 1,

which gives:

(x1)21+y2=0(x1)2+y2=1.(x - 1)^2 - 1 + y^2 = 0 \quad \Rightarrow \quad (x - 1)^2 + y^2 = 1.

This is the standard equation of a circle with center at ( (1, 0) ) and radius ( 1 ) in the complex plane.

Visualizations:

[Table omitted in web version]

Problem 1.31: Equilateral Triangle on the Unit Circle

Section titled “Problem 1.31: Equilateral Triangle on the Unit Circle”

Problem. Given three complex numbers ( z_1, z_2, z_3 ) such that ( |z_1| = |z_2| = |z_3| = 1 ) and ( z_1 + z_2 + z_3 = 0 ), show that these numbers are vertices of an equilateral triangle inscribed in the unit circle with center at the origin.

Strategy: Use the fact that the sum of three unit complex numbers equals zero to show they must be the cube roots of unity (rotated), which form an equilateral triangle. Verify that the angles differ by 2π/32\pi/3 and the sum condition is satisfied.

Solution: Since ( |z_i| = 1 ), each ( z_i = e^{i\theta_i} ) lies on the unit circle. Given ( z_1 + z_2 + z_3 = 0 ), we need to show they form an equilateral triangle. Consider the angles ( \theta_1, \theta_2, \theta_3 ). The sum condition implies:

eiθ1+eiθ2+eiθ3=0.e^{i\theta_1} + e^{i\theta_2} + e^{i\theta_3} = 0.

For three points on the unit circle to form an equilateral triangle, their arguments must differ by ( 120^\circ = \frac{2\pi}{3} ). Assume:

z1=eiθ,z2=ei(θ+2π3),z3=ei(θ+4π3).z_1 = e^{i\theta}, \quad z_2 = e^{i(\theta + \frac{2\pi}{3})}, \quad z_3 = e^{i(\theta + \frac{4\pi}{3})}.

Check the sum:

eiθ+ei(θ+2π3)+ei(θ+4π3)=eiθ(1+ei2π3+ei4π3).e^{i\theta} + e^{i(\theta + \frac{2\pi}{3})} + e^{i(\theta + \frac{4\pi}{3})} = e^{i\theta} \left( 1 + e^{i\frac{2\pi}{3}} + e^{i\frac{4\pi}{3}} \right).

Since ( e^{i\frac{2\pi}{3}} = -\frac{1}{2} + i\frac{\sqrt{3}}{2} ), ( e^{i\frac{4\pi}{3}} = -\frac{1}{2} - i\frac{\sqrt{3}}{2} ), we have:

1+ei2π3+ei4π3=1+(12+i32)+(12i32)=0.1 + e^{i\frac{2\pi}{3}} + e^{i\frac{4\pi}{3}} = 1 + \left(-\frac{1}{2} + i\frac{\sqrt{3}}{2}\right) + \left(-\frac{1}{2} - i\frac{\sqrt{3}}{2}\right) = 0.

The angles ( \theta, \theta + \frac{2\pi}{3}, \theta + \frac{4\pi}{3} ) are spaced ( \frac{2\pi}{3} ) apart, forming an equilateral triangle. Any three points with ( |z_i| = 1 ) and sum zero are rotations of the cube roots of unity, ensuring an equilateral triangle.

Visualization:

[TikZ diagram omitted in web version]

This visualization shows the equilateral triangle formed by the cube roots of unity on the unit circle. The green vectors show how the sum of the three complex numbers equals zero, and the orange angle markers show the 120°120° spacing between vertices.

Problem 1.32: Inequality with Complex Numbers

Section titled “Problem 1.32: Inequality with Complex Numbers”

Problem. If ( a ) and ( b ) are complex numbers, prove:

  1. ( |a - b|^2 \leq (1 + |a|^2)(1 + |b|^2) ),
  2. If ( a \neq 0 ), then ( |a + b| = |a| + |b| ) if and only if ( \frac{b}{a} ) is real and nonnegative.

Strategy: For part (a), we’ll expand both sides and show that the difference is non-negative. For part (b), we’ll use the fact that equality in the triangle inequality occurs when the complex numbers are collinear and point in the same direction.

Solution:

  1. Compute:
ab2=(ab)(ab)=a2+b2abab.|a - b|^2 = (a - b)(\overline{a - b}) = |a|^2 + |b|^2 - a\overline{b} - \overline{a}b.

Consider the right-hand side:

(1+a2)(1+b2)=1+a2+b2+a2b2.(1 + |a|^2)(1 + |b|^2) = 1 + |a|^2 + |b|^2 + |a|^2 |b|^2.

Evaluate:

(1+a2)(1+b2)ab2=1+ab2+ab+ab=1+ab2+2(ab).(1 + |a|^2)(1 + |b|^2) - |a - b|^2 = 1 + |a b|^2 + a\overline{b} + \overline{a}b = 1 + |a b|^2 + 2\Re(a\overline{b}).

Since ( |a b|^2 \geq 0 ), ( \Re(a\overline{b}) \geq -|a b| ):

1+ab2+2(ab)1+ab22ab=(1ab)20.1 + |a b|^2 + 2\Re(a\overline{b}) \geq 1 + |a b|^2 - 2|a b| = (1 - |a b|)^2 \geq 0.

Thus, ( |a - b|^2 \leq (1 + |a|^2)(1 + |b|^2) ).

  1. For ( |a + b| = |a| + |b| ), the triangle inequality requires ( a, b ) collinear in the same direction. Let ( b = ka ), ( k \in \mathbb{R}_{\geq 0} ):
a+b=a+ka=a(1+k)=a+b.|a + b| = |a + ka| = |a|(1 + k) = |a| + |b|.

Thus, ( \frac{b}{a} = k \geq 0 ). Conversely, if ( |a + b| = |a| + |b| ), then ( a\overline{b} + \overline{a}b = 2|a||b| ), so ( \frac{b}{a} ) is real and nonnegative.

Problem 1.33: Equality Condition for Complex Difference

Section titled “Problem 1.33: Equality Condition for Complex Difference”

Problem. If ( a ) and ( b ) are complex numbers, prove that

ab=1ab|a - b| = |1 - \overline{a}b|

if and only if ( |a| = 1 ) or ( |b| = 1 ). For which ( a ) and ( b ) is the inequality ( |a - b| < |1 - \overline{a}b| ) valid?

Strategy: We will compute the difference ab21ab2|a - b|^2 - |1 - \overline{a}b|^2 and show that it factors as (r21)(s21)(r^2 - 1)(s^2 - 1) where r=ar = |a| and s=bs = |b|. This will allow us to determine when equality holds and when the inequality is valid.

Solution: Let ( |a| = r ), ( |b| = s ). Compute:

ab2=r2+s2abab,1ab2=1+r2s2abab.|a - b|^2 = r^2 + s^2 - a\overline{b} - \overline{a}b, \quad |1 - \overline{a}b|^2 = 1 + r^2 s^2 - a\overline{b} - \overline{a}b.

Thus:

ab21ab2=r2+s21r2s2=(r21)(s21).|a - b|^2 - |1 - \overline{a}b|^2 = r^2 + s^2 - 1 - r^2 s^2 = (r^2 - 1)(s^2 - 1).

Equality holds when:

(r21)(s21)=0    r=1 or s=1.(r^2 - 1)(s^2 - 1) = 0 \implies r = 1 \text{ or } s = 1.

For the inequality:

(r21)(s21)<0    (r2<1 and s2>1) or (r2>1 and s2<1).(r^2 - 1)(s^2 - 1) < 0 \implies (r^2 < 1 \text{ and } s^2 > 1) \text{ or } (r^2 > 1 \text{ and } s^2 < 1).

Thus, equality holds if ( |a| = 1 ) or ( |b| = 1 ); the inequality holds when one modulus is less than 1 and the other is greater than 1.

Problem. If ( a ) and ( c ) are real constants, ( b ) is complex, show that the equation

azz+bz+bz+c=0(a0,z=x+iy)az\overline{z} + bz + \overline{b} \overline{z} + c = 0 \qquad (a \ne 0, z = x + iy)

represents a circle in the ( xy )-plane.

Strategy: We will substitute z=x+iyz = x + iy and z=xiy\overline{z} = x - iy into the equation, then use the fact that zz=x2+y2z\overline{z} = x^2 + y^2 and bz+bz=2(bz)bz + \overline{b}\overline{z} = 2\Re(bz) to show that the equation reduces to the general form of a circle.

Solution: Let ( z = x + iy ), ( \overline{z} = x - iy ), then ( z \overline{z} = x^2 + y^2 ), ( bz + \overline{b} \overline{z} = 2 \Re(b z) ). Hence the equation becomes:

a(x2+y2)+2(bz)+c=0.a(x^2 + y^2) + 2 \Re(b z) + c = 0.

This is the general form of a circle in ( \mathbb{R}^2 ).

Problem 1.35: Argument of a Complex Number via Arctangent

Section titled “Problem 1.35: Argument of a Complex Number via Arctangent”

Problem. Recall the definition of the inverse tangent: given a real number ( t ), ( \tan^{-1}(t) ) is the unique real number ( \theta ) satisfying:

π2<θ<π2,andtanθ=t.-\frac{\pi}{2} < \theta < \frac{\pi}{2}, \quad \text{and} \quad \tan \theta = t.

If ( z = x + iy ), show that:

  1. ( \arg(z) = \tan^{-1}\left( \frac{y}{x} \right) ), if ( x > 0 ),
  2. ( \arg(z) = \tan^{-1}\left( \frac{y}{x} \right) + \pi ), if ( x < 0 ), ( y \geq 0 ),
  3. ( \arg(z) = \tan^{-1}\left( \frac{y}{x} \right) - \pi ), if ( x < 0 ), ( y < 0 ),
  4. ( \arg(z) = \frac{\pi}{2} ), if ( x = 0, y > 0 ); \quad ( \arg(z) = -\frac{\pi}{2} ), if ( x = 0, y < 0 ).

Strategy: We will use the relationship between the argument of a complex number and the quadrant it lies in. The principal value of tan1\tan^{-1} gives angles in (π/2,π/2](-\pi/2, \pi/2], so we need to adjust for different quadrants to get the correct argument in (π,π](-\pi, \pi].

Solution: For ( z = x + iy ), ( \arg(z) ) is the angle ( \theta \in (-\pi, \pi] ) such that ( z = |z| e^{i\theta} ).

  1. If ( x > 0 ), ( z ) is in Quadrant I or IV, and ( \tan \theta = \frac{y}{x} ), so ( \theta = \tan^{-1}\left( \frac{y}{x} \right) ).
  2. If ( x < 0 ), ( y \geq 0 ), ( z ) is in Quadrant II. ( \tan^{-1}\left( \frac{y}{x} \right) \in (-\frac{\pi}{2}, 0] ), so add ( \pi ) to get ( \theta \in (\frac{\pi}{2}, \pi] ).
  3. If ( x < 0 ), ( y < 0 ), ( z ) is in Quadrant III. ( \tan^{-1}\left( \frac{y}{x} \right) \in (0, \frac{\pi}{2}] ), so subtract ( \pi ) to get ( \theta \in (-\pi, -\frac{\pi}{2}] ).
  4. If ( x = 0 ), ( z = iy ). If ( y > 0 ), ( \theta = \frac{\pi}{2} ); if ( y < 0 ), ( \theta = -\frac{\pi}{2} ).

Problem 1.36: Pseudo-Ordering on Complex Numbers

Section titled “Problem 1.36: Pseudo-Ordering on Complex Numbers”

Problem. Define the following pseudo-ordering on complex numbers: z1<z2z_1 < z_2 if z1<z2|z_1| < |z_2|, or if z1=z2|z_1|=|z_2| and arg(z1)<arg(z2)\arg(z_1) < \arg(z_2). Which of Axioms 6,7,8,9 are satisfied by this relation?

Strategy: We will examine each axiom individually, testing whether the pseudo-ordering satisfies the properties of trichotomy, translation invariance, multiplication invariance, and transitivity. We’ll provide counterexamples where axioms fail.

  • Axiom 6 (Trichotomy): For any z1,z2Cz_1, z_2 \in \mathbb{C}, we can compare their moduli. Exactly one of z1<z2|z_1| < |z_2|, z1>z2|z_1| > |z_2|, or z1=z2|z_1| = |z_2| holds. If z1=z2|z_1| = |z_2|, we compare their principal arguments, for which trichotomy holds on (π,π](-\pi, \pi]. Thus, exactly one of z1<z2z_1 < z_2, z2<z1z_2 < z_1, or z1=z2z_1 = z_2 is true. This axiom is satisfied.

  • Axiom 9 (Transitivity): If z1<z2z_1 < z_2 and z2<z3z_2 < z_3, the transitivity of the << relation on the real numbers for both the moduli and the arguments ensures that z1<z3z_1 < z_3. This axiom is satisfied.

  • Axiom 7 (Translation Invariance): This axiom states that if z1<z2z_1 < z_2, then z1+z<z2+zz_1 + z < z_2 + z for any zCz \in \mathbb{C}. This axiom is not satisfied.

    Counterexample: Let z1=1z_1 = 1 and z2=2z_2 = 2. According to the ordering, z1<z2z_1 < z_2 because z1=1<z2=2|z_1|=1 < |z_2|=2. Now, let z=2z = -2. Then z1+z=1+(2)=1z_1 + z = 1 + (-2) = -1. And z2+z=2+(2)=0z_2 + z = 2 + (-2) = 0. We must compare z1+z=1z_1+z = -1 and z2+z=0z_2+z=0. We have 1=1|-1|=1 and 0=0|0|=0. Since 0<1|0| < |-1|, we have 0<10 < -1 in this pseudo-ordering. So, z2+z<z1+zz_2 + z < z_1 + z. The order relation was reversed, which violates the axiom.

  • Axiom 8 (Multiplication): This axiom states that if z1<z2z_1 < z_2 and z>0z > 0, then z1z<z2zz_1 z < z_2 z. Let us define z>0z>0 to mean 0<z0<z. This holds for any z0z \neq 0. This axiom is also not satisfied.

    Counterexample: Let z1=eiπ=1z_1 = e^{i\pi} = -1 and z2=eiπ/2=iz_2 = e^{-i\pi/2} = -i. We have z1=z2=1|z_1| = |z_2| = 1. The arguments are arg(z1)=π\arg(z_1) = \pi and arg(z2)=π/2\arg(z_2) = -\pi/2. Since π/2<π-\pi/2 < \pi, we have z2<z1z_2 < z_1. Now, let z=iz = i. Since i0i \neq 0, zz is a “positive” number under this definition. Then z1z=(1)(i)=iz_1 z = (-1)(i) = -i. And z2z=(i)(i)=1z_2 z = (-i)(i) = 1. We must compare z1z=iz_1 z = -i and z2z=1z_2 z = 1. We have i=1|-i|=1 and 1=1|1|=1. The arguments are arg(i)=π/2\arg(-i) = -\pi/2 and arg(1)=0\arg(1) = 0. Since π/2<0-\pi/2 < 0, we have i<1-i < 1. So, z1z<z2zz_1 z < z_2 z. The order relation was reversed from z2<z1z_2 < z_1 to z1z<z2zz_1 z < z_2 z. The axiom is violated.

Conclusion: Axioms 6 and 9 are satisfied; Axiom 7 and 8 is not applicable.

Solution: The pseudo-ordering on complex numbers is defined by z1<z2z_1 < z_2 if either z1<z2|z_1| < |z_2| or z1=z2|z_1| = |z_2| and arg(z1)<arg(z2)\arg(z_1) < \arg(z_2).

Axiom 6 (Trichotomy): For any z1,z2Cz_1, z_2 \in \mathbb{C}, exactly one of the following holds:

  1. z1<z2|z_1| < |z_2|, in which case z1<z2z_1 < z_2
  2. z1>z2|z_1| > |z_2|, in which case z2<z1z_2 < z_1
  3. z1=z2|z_1| = |z_2|, in which case we compare arguments: \begin{enumerate}[label=(\roman*)]
  4. arg(z1)<arg(z2)\arg(z_1) < \arg(z_2), so z1<z2z_1 < z_2
  5. arg(z1)>arg(z2)\arg(z_1) > \arg(z_2), so z2<z1z_2 < z_1
  6. arg(z1)=arg(z2)\arg(z_1) = \arg(z_2), so z1=z2z_1 = z_2

\end{enumerate} Therefore, Axiom 6 is satisfied.

Axiom 7 (Translation Invariance): This axiom is not satisfied. Consider z1=1z_1 = 1 and z2=2z_2 = 2. We have z1<z2z_1 < z_2 since 1=1<2=2|1| = 1 < |2| = 2. However, with z=2z = -2, we get z1+z=1z_1 + z = -1 and z2+z=0z_2 + z = 0. Since 0=0<1=1|0| = 0 < |-1| = 1, we have z2+z<z1+zz_2 + z < z_1 + z, reversing the order.

Axiom 8 (Multiplication Invariance): This axiom is not satisfied. Consider z1=1=eiπz_1 = -1 = e^{i\pi} and z2=i=eiπ/2z_2 = -i = e^{-i\pi/2}. Since z1=z2=1|z_1| = |z_2| = 1 and arg(z2)=π/2<π=arg(z1)\arg(z_2) = -\pi/2 < \pi = \arg(z_1), we have z2<z1z_2 < z_1. However, with z=iz = i, we get z1z=iz_1 z = -i and z2z=1z_2 z = 1. Since i=1=1|-i| = |1| = 1 and arg(i)=π/2<0=arg(1)\arg(-i) = -\pi/2 < 0 = \arg(1), we have z1z<z2zz_1 z < z_2 z, again reversing the order.

Axiom 9 (Transitivity): If z1<z2z_1 < z_2 and z2<z3z_2 < z_3, then either:

  1. z1<z2<z3|z_1| < |z_2| < |z_3|, so z1<z3|z_1| < |z_3| and z1<z3z_1 < z_3
  2. z1<z2=z3|z_1| < |z_2| = |z_3|, so z1<z3|z_1| < |z_3| and z1<z3z_1 < z_3
  3. z1=z2<z3|z_1| = |z_2| < |z_3|, so z1<z3|z_1| < |z_3| and z1<z3z_1 < z_3
  4. z1=z2=z3|z_1| = |z_2| = |z_3| and arg(z1)<arg(z2)<arg(z3)\arg(z_1) < \arg(z_2) < \arg(z_3), so arg(z1)<arg(z3)\arg(z_1) < \arg(z_3) and z1<z3z_1 < z_3

Therefore, Axiom 9 is satisfied.

In summary, Axioms 6 and 9 are satisfied, while Axioms 7 and 8 are not satisfied.

Problem 1.37: Order Axioms and Lexicographic Ordering on R2\mathbb{R}^2

Section titled “Problem 1.37: Order Axioms and Lexicographic Ordering on R2\mathbb{R}^2R2”

Problem. Define a pseudo-ordering on ordered pairs ((x_1, y_1) < (x_2, y_2)) if either

  1. (x_1 < x_2), or
  2. (x_1 = x_2) and (y_1 < y_2).

Which of Axioms 6, 7, 8, 9 are satisfied by this relation?

Strategy: We will examine each axiom for the lexicographic ordering on R2\mathbb{R}^2. This ordering compares first coordinates, then second coordinates if the first coordinates are equal, which should preserve most of the standard ordering properties.

Solution:

  • Axiom 6: Trichotomy. For any ( (x_1, y_1), (x_2, y_2) ), if ( x_1 < x_2 ), then ( (x_1, y_1) < (x_2, y_2) ); if ( x_1 > x_2 ), then ( (x_2, y_2) < (x_1, y_1) ); if ( x_1 = x_2 ), compare ( y_1, y_2 ). Exactly one holds. Satisfied.
  • Axiom 7: Translation Invariance. If ( (x_1, y_1) < (x_2, y_2) ), add ( (u, v) ): if ( x_1 < x_2 ), then ( x_1 + u < x_2 + u ); if ( x_1 = x_2 ), then ( y_1 < y_2 \implies y_1 + v < y_2 + v ). Satisfied.
  • Axiom 8: Multiplication. Not applicable, as ( \mathbb{R}^2 ) lacks scalar multiplication.
  • Axiom 9: Transitivity. If ( (x_1, y_1) < (x_2, y_2) ), ( (x_2, y_2) < (x_3, y_3) ), lexicographic order ensures ( (x_1, y_1) < (x_3, y_3) ). Satisfied.

Conclusion: Axioms 6, 7, and 9 are satisfied; Axiom 8 is not applicable.

Problem 1.38: Argument of a Quotient Using Theorem 1.48

Section titled “Problem 1.38: Argument of a Quotient Using Theorem 1.48”

Problem. State and prove a theorem analogous to Theorem 1.48, expressing ( \arg\left( \frac{z_1}{z_2} \right) ) in terms of ( \arg(z_1) ) and ( \arg(z_2) ).

Strategy: We will use the fact that z1z2=z1z21\frac{z_1}{z_2} = z_1 z_2^{-1} and apply Theorem 1.48 to the product, using the property that arg(z21)=arg(z2)\arg(z_2^{-1}) = -\arg(z_2).

Solution: Theorem: If ( z_1, z_2 \neq 0 ), then:

arg(z1z2)=arg(z1)arg(z2)+2πn(z1,z21),\arg\left( \frac{z_1}{z_2} \right) = \arg(z_1) - \arg(z_2) + 2\pi n(z_1, z_2^{-1}),

where ( n(z_1, z_2^{-1}) ) adjusts the argument to ( (-\pi, \pi] ).

Solution: Since ( \frac{z_1}{z_2} = z_1 z_2^{-1} ), and ( \arg(z_2^{-1}) = -\arg(z_2) ), apply Theorem 1.48:

arg(z1z21)=arg(z1)+arg(z21)+2πn(z1,z21)=arg(z1)arg(z2)+2πn(z1,z21).\begin{aligned} \arg(z_1 z_2^{-1}) =& \arg(z_1) + \arg(z_2^{-1}) + 2\pi n(z_1, z_2^{-1}) \\ =& \arg(z_1) - \arg(z_2) + 2\pi n(z_1, z_2^{-1}). \end{aligned}

Problem 1.39: Logarithm of a Quotient Using Theorem 1.54

Section titled “Problem 1.39: Logarithm of a Quotient Using Theorem 1.54”

Problem. State and prove a theorem analogous to Theorem 1.54, expressing ( \log\left( \frac{z_1}{z_2} \right) ) in terms of ( \log(z_1) ) and ( \log(z_2) ).

Strategy: We will use the fact that z1z2=z1z21\frac{z_1}{z_2} = z_1 z_2^{-1} and apply Theorem 1.54 to the product, using the property that log(z21)=log(z2)\log(z_2^{-1}) = -\log(z_2).

Solution: Theorem: If ( z_1, z_2 \neq 0 ), then:

log(z1z2)=logz1logz2+2πin(z1,z21).\log\left( \frac{z_1}{z_2} \right) = \log z_1 - \log z_2 + 2\pi i n(z_1, z_2^{-1}).

Solution: Since ( \frac{z_1}{z_2} = z_1 z_2^{-1} ), apply Theorem 1.54:

log(z1z21)=logz1+log(z21)+2πin(z1,z21)=logz1logz2+2πin(z1,z21).\begin{aligned} \log(z_1 z_2^{-1}) =& \log z_1 + \log(z_2^{-1}) + 2\pi i n(z_1, z_2^{-1}) \\ =& \log z_1 - \log z_2 + 2\pi i n(z_1, z_2^{-1}). \end{aligned}

Problem 1.40: Roots of Unity and Polynomial Identity

Section titled “Problem 1.40: Roots of Unity and Polynomial Identity”

Problem. Prove that the ( n )th roots of 1 are given by ( \alpha, \alpha^2, \ldots, \alpha^n ), where ( \alpha = e^{2\pi i/n} ), and that these roots ( \ne 1 ) satisfy the equation

1+x+x2++xn1=0.1 + x + x^2 + \cdots + x^{n-1} = 0.

Strategy: We will use the fact that the nnth roots of unity are the solutions to xn1=0x^n - 1 = 0, and use the factorization xn1x1=1+x+x2++xn1\frac{x^n - 1}{x - 1} = 1 + x + x^2 + \cdots + x^{n-1} to show that all roots except x=1x = 1 satisfy the given equation.

Solution: Let ( \alpha = e^{2\pi i/n} ). Then ( \alpha^n = 1 ), so it’s a root of ( x^n - 1 = 0 ). Also,

1αn1α=01+α++αn1=0for α1.\frac{1 - \alpha^n}{1 - \alpha} = 0 \Rightarrow 1 + \alpha + \cdots + \alpha^{n-1} = 0 \quad \text{for } \alpha \ne 1.

Problem 1.41: Inequalities and Boundedness of cos z

Section titled “Problem 1.41: Inequalities and Boundedness of cos z”

Problem.

  1. Prove that ( |z^i| < e^{\pi} ) for all complex ( z \ne 0 ).
  2. Prove that there is no constant ( M > 0 ) such that ( |\cos z| < M ) for all complex ( z ).

Strategy: For part (a), we’ll use the definition zi=eilogzz^i = e^{i\log z} and analyze the modulus in terms of the argument. For part (b), we’ll use the fact that cos(iy)=coshy\cos(iy) = \cosh y which grows exponentially as yy \to \infty.

Solution:

  1. For ( z = re^{i\theta} ), ( z^i = e^{i(\ln r + i\theta)} = e^{-\theta} e^{i \ln r} ), so ( |z^i| = e^{-\theta} ). Since ( \theta \in (-\pi, \pi] ), ( |z^i| \leq e^{\pi} ), strict unless ( \theta = -\pi ).
  2. For ( z = iy ), ( \cos(iy) = \cosh y ), which is unbounded as ( |y| \to \infty ). Thus, no ( M > 0 ) exists.

Problem 1.42: Complex Exponential via Real and Imaginary Parts

Section titled “Problem 1.42: Complex Exponential via Real and Imaginary Parts”

Problem. If ( w = u + iv ), where ( u ) and ( v ) are real, show that

zw=eulogzvarg(z)ei[vlogz+uarg(z)].z^w = e^{u \log |z| - v \arg(z)} \cdot e^{i[v \log |z| + u \arg(z)]}.

Strategy: We will use the definition zw=ewlogzz^w = e^{w \log z} and expand the product (u+iv)(logz+iargz)(u + iv)(\log |z| + i \arg z) to separate the real and imaginary parts.

Solution: For ( z^w = e^{w \log z} ), where ( \log z = \log |z| + i \arg z ):

wlogz=(u+iv)(logz+iargz)=(ulogzvargz)+i(vlogz+uargz).w \log z = (u + iv)(\log |z| + i \arg z) = (u \log |z| - v \arg z) + i(v \log |z| + u \arg z).

Thus:

zw=eulogzvargzei(vlogz+uargz).z^w = e^{u \log |z| - v \arg z} e^{i(v \log |z| + u \arg z)}.

Problem 1.43: Logarithmic Identities for Complex Powers

Section titled “Problem 1.43: Logarithmic Identities for Complex Powers”

Problem.

  1. Prove that ( \log(z^w) = w \log z + 2\pi i n ), where ( n ) is an integer.
  2. Prove that ( (z^w)^\alpha = z^{w\alpha} e^{2\pi i n \alpha} ), where ( n ) is an integer.

Strategy: For part (a), we’ll use the definition zw=ewlogzz^w = e^{w \log z} and the fact that log(ew)=w+2πin\log(e^w) = w + 2\pi i n. For part (b), we’ll use the result from part (a) and the definition of complex exponentiation.

Solution:

  1. Since ( z^w = e^{w \log z} ):
log(zw)=log(ewlogz)=wlogz+2πin.\log(z^w) = \log(e^{w \log z}) = w \log z + 2\pi i n.
  1. Compute:
(zw)α=eαlog(zw)=eα(wlogz+2πin)=zwαe2πinα.(z^w)^\alpha = e^{\alpha \log(z^w)} = e^{\alpha(w \log z + 2\pi i n)} = z^{w\alpha} e^{2\pi i n \alpha}.

Problem 1.44: Conditions for De Moivre’s Formula

Section titled “Problem 1.44: Conditions for De Moivre’s Formula”

Problem.

  1. If ( \theta ) and ( a ) are real numbers, ( -\pi < \theta \leq +\pi ), prove that
(cosθ+isinθ)a=cos(aθ)+isin(aθ).(\cos \theta + i \sin \theta)^a = \cos(a\theta) + i \sin(a\theta).
  1. Show that, in general, the restriction ( -\pi < \theta \leq +\pi ) is necessary in (i) by taking ( \theta = -\pi ), ( a = \tfrac{1}{2} ).
  2. If ( a ) is an integer, show that the formula in (i) holds without any restriction on ( \theta ). In this case it is known as De Moivre’s theorem.

Strategy: We will use the fact that cosθ+isinθ=eiθ\cos \theta + i \sin \theta = e^{i\theta} and the definition of complex exponentiation. For part (b), we’ll provide a specific counterexample. For part (c), we’ll use the fact that integer powers don’t have branch cut issues.

Solution:

  1. Since ( \cos \theta + i \sin \theta = e^{i\theta} ):
(cosθ+isinθ)a=(eiθ)a=eiaθ=cos(aθ)+isin(aθ).(\cos \theta + i \sin \theta)^a = (e^{i\theta})^a = e^{i a \theta} = \cos(a\theta) + i \sin(a\theta).
  1. For ( \theta = -\pi ), ( a = \frac{1}{2} ):
(1)1/2=i,butcos(π2)+isin(π2)=i.(-1)^{1/2} = i, \quad \text{but} \quad \cos\left(\frac{-\pi}{2}\right) + i \sin\left(\frac{-\pi}{2}\right) = -i.

The restriction ensures the principal branch.

  1. For integer ( a ), ( (e^{i\theta})^a = e^{i a \theta} ), and multiples of ( 2\pi ) cancel, so the formula holds for all ( \theta ).

Problem 1.45: Deriving Trigonometric Identities from De Moivre’s Theorem

Section titled “Problem 1.45: Deriving Trigonometric Identities from De Moivre’s Theorem”

Problem. Use De Moivre’s theorem (Exercise 1.44) to derive the trigonometric identities

sin3θ=3cos2θsinθsin3θ,\sin 3\theta = 3 \cos^2 \theta \sin \theta - \sin^3 \theta, cos3θ=cos3θ3cosθsin2θ,\cos 3\theta = \cos^3 \theta - 3 \cos \theta \sin^2 \theta,

valid for real ( \theta ). Are these valid when ( \theta ) is complex?

Strategy: We will use De Moivre’s theorem to expand (cosθ+isinθ)3(\cos \theta + i \sin \theta)^3, then equate the real and imaginary parts to obtain the desired identities. Since cosz\cos z and sinz\sin z are analytic functions, these identities extend to complex θ\theta.

Solution: By De Moivre’s theorem:

(cosθ+isinθ)3=cos3θ+isin3θ.(\cos \theta + i \sin \theta)^3 = \cos 3\theta + i \sin 3\theta.

Expand:

cos3θ+3icos2θsinθ3cosθsin2θisin3θ.\cos^3 \theta + 3i \cos^2 \theta \sin \theta - 3 \cos \theta \sin^2 \theta - i \sin^3 \theta.

Equate parts:

cos3θ=cos3θ3cosθsin2θ,sin3θ=3cos2θsinθsin3θ.\cos 3\theta = \cos^3 \theta - 3 \cos \theta \sin^2 \theta, \quad \sin 3\theta = 3 \cos^2 \theta \sin \theta - \sin^3 \theta.

These hold for complex ( \theta ), as ( \cos z ) and ( \sin z ) are analytic.

Problem. Define ( \tan z = \frac{\sin z}{\cos z} ), and show that for ( z = x + iy ),

tanz=sin2x+isinh2ycos2x+cosh2y.\tan z = \frac{\sin 2x + i \sinh 2y}{\cos 2x + \cosh 2y}.

Strategy: We will use the expressions for sinz\sin z and cosz\cos z in terms of real and imaginary parts, then rationalize the denominator by multiplying by the complex conjugate and simplify using trigonometric and hyperbolic identities.

Solution: For ( z = x + iy ):

sinz=sinxcoshy+icosxsinhy,cosz=cosxcoshyisinxsinhy.\sin z = \sin x \cosh y + i \cos x \sinh y, \quad \cos z = \cos x \cosh y - i \sin x \sinh y.

Compute:

tanz=sinxcoshy+icosxsinhycosxcoshyisinxsinhy.\tan z = \frac{\sin x \cosh y + i \cos x \sinh y}{\cos x \cosh y - i \sin x \sinh y}.

Multiply by the conjugate of the denominator:

N=(sinxcoshy+icosxsinhy)(cosxcoshy+isinxsinhy)=sin2x+isinh2y,N = (\sin x \cosh y + i \cos x \sinh y)(\cos x \cosh y + i \sin x \sinh y) = \sin 2x + i \sinh 2y, D=(cosxcoshy)2+(sinxsinhy)2=12(cos2x+cosh2y).D = (\cos x \cosh y)^2 + (\sin x \sinh y)^2 = \frac{1}{2}(\cos 2x + \cosh 2y).

Thus:

tanz=sin2x+isinh2ycos2x+cosh2y.\tan z = \frac{\sin 2x + i \sinh 2y}{\cos 2x + \cosh 2y}.

Problem. Let ( w ) be a complex number. If ( w \ne \pm 1 ), show that there exist two values ( z = x + iy ) with ( \cos z = w ) and ( -\pi < x \leq \pi ). Find such ( z ) when ( w = i ) and ( w = 2 ).

Strategy: We will use the expression for cosz\cos z in terms of real and imaginary parts, then solve the resulting system of equations for xx and yy. We’ll provide specific solutions for the given values of ww.

Solution: For ( z = x + iy ), ( \cos z = \cos x \cosh y - i \sin x \sinh y = w = u + iv ). Solve:

cosxcoshy=u,sinxsinhy=v.\cos x \cosh y = u, \quad -\sin x \sinh y = v.

Square and add:

sin2x=sinh2y+1u2v2.\sin^2 x = \sinh^2 y + 1 - u^2 - v^2.

Since ( w \neq \pm 1 ), solutions exist, with two ( x ) in ( (-\pi, \pi] ).

Case 1: ( w = i ). ( u = 0 ), ( v = 1 ):

cosxcoshy=0    x=±π2.\cos x \cosh y = 0 \implies x = \pm \frac{\pi}{2}.

For ( x = \frac{\pi}{2} ), ( \sinh y = -1 \implies y = -\ln(1 + \sqrt{2}) ). For ( x = -\frac{\pi}{2} ), ( \sinh y = 1 \implies y = \ln(1 + \sqrt{2}) ). Solutions: ( z_1 = \frac{\pi}{2} - i \ln(1 + \sqrt{2}) ), ( z_2 = -\frac{\pi}{2} + i \ln(1 + \sqrt{2}) ).

Case 2: ( w = 2 ). ( u = 2 ), ( v = 0 ):

cosxcoshy=2,sinxsinhy=0.\cos x \cosh y = 2, \quad \sin x \sinh y = 0.

Thus, ( x = 0 ), ( \cosh y = 2 \implies y = \pm \ln(2 + \sqrt{3}) ). Solutions: ( z_1 = i \ln(2 + \sqrt{3}) ), ( z_2 = -i \ln(2 + \sqrt{3}) ).

Visualization:

[TikZ diagram omitted in web version]

This visualization shows the solutions to cosz=w\cos z = w for w=iw = i (red points) and w=2w = 2 (blue points) within the strip π<xπ-\pi < x \leq \pi. The gray shaded region represents the strip where we seek solutions.

Problem 1.48: Lagrange’s Identity and the Cauchy–Schwarz Inequality

Section titled “Problem 1.48: Lagrange’s Identity and the Cauchy–Schwarz Inequality”

Problem. Prove Lagrange’s identity for complex numbers:

k=1nakbk2=(k=1nak2)(k=1nbk2)1k<jnakbjajbk2.\left| \sum_{k=1}^n a_k \overline{b_k} \right|^2 = \left( \sum_{k=1}^n |a_k|^2 \right) \left( \sum_{k=1}^n |b_k|^2 \right) - \sum_{1 \leq k < j \leq n} |a_k \overline{b_j} - a_j \overline{b_k}|^2.

Use this to deduce a Cauchy–Schwarz inequality for complex numbers.

Strategy: We will expand both sides of the identity and show they are equal by careful algebraic manipulation. Since the right-hand side includes a sum of squares of absolute values, it is non-negative, which will immediately give us the Cauchy-Schwarz inequality.

Solution: We want to prove the identity:

k=1nakbk2=(k=1nak2)(j=1nbj2)1k<jnakbjajbk2.\left| \sum_{k=1}^n a_k \overline{b_k} \right|^2 = \left( \sum_{k=1}^n |a_k|^2 \right) \left( \sum_{j=1}^n |b_j|^2 \right) - \sum_{1 \leq k < j \leq n} |a_k \overline{b_j} - a_j \overline{b_k}|^2.

It is easier to prove the equivalent formulation:

(k=1nak2)(j=1nbj2)=k=1nakbk2+1k<jnakbjajbk2.\left( \sum_{k=1}^n |a_k|^2 \right) \left( \sum_{j=1}^n |b_j|^2 \right) = \left| \sum_{k=1}^n a_k \overline{b_k} \right|^2 + \sum_{1 \leq k < j \leq n} |a_k \overline{b_j} - a_j \overline{b_k}|^2.

Let’s expand the left-hand side (LHS):

LHS=(k=1nakak)(j=1nbjbj)=k=1nj=1nakakbjbj=k=jak2bj2+kjak2bj2=k=1nak2bk2+1k<jn(ak2bj2+aj2bk2)\begin{aligned} \text{LHS} &= \left( \sum_{k=1}^n a_k \overline{a_k} \right) \left( \sum_{j=1}^n b_j \overline{b_j} \right) = \sum_{k=1}^n \sum_{j=1}^n a_k \overline{a_k} b_j \overline{b_j} \\ &= \sum_{k=j} a_k^2 b_j^2 + \sum_{k \neq j} a_k^2 b_j^2 \\ &= \sum_{k=1}^n a_k^2 b_k^2 + \sum_{1 \leq k < j \leq n} (a_k^2 b_j^2 + a_j^2 b_k^2) \end{aligned}

Now, let’s expand the right-hand side (RHS). The first term is:

k=1nakbk2=(k=1nakbk)(j=1najbj)=(k=1nakbk)(j=1najbj)=k=jakbkajbj+kjakbkajbj=k=1nak2bk2+21k<jnakbkajbj\begin{aligned} \left| \sum_{k=1}^n a_k \overline{b_k} \right|^2 &= \left(\sum_{k=1}^n a_k \overline{b_k}\right) \overline{\left(\sum_{j=1}^n a_j \overline{b_j}\right)} = \left(\sum_{k=1}^n a_k \overline{b_k}\right) \left(\sum_{j=1}^n \overline{a_j} b_j\right) \\ &= \sum_{k=j} a_k b_k a_j b_j + \sum_{k \neq j} a_k b_k a_j b_j \\ &= \sum_{k=1}^n a_k^2 b_k^2 + 2 \sum_{1 \leq k < j \leq n} a_k b_k a_j b_j \end{aligned}

The second term on the RHS is:

1k<jnakbjajbk2=1k<jn(akbjajbk)(akbjajbk)=1k<jn(akbjajbk)(akbjajbk)=1k<jn(akbjakbjakbjajbkajbkakbj+ajbkajbk)=1k<jn(ak2bj2akbkajbjakbkajbj+aj2bk2)\begin{aligned} &\sum_{1 \leq k < j \leq n} |a_k \overline{b_j} - a_j \overline{b_k}|^2 \\ =& \sum_{1 \leq k < j \leq n} (a_k \overline{b_j} - a_j \overline{b_k}) \overline{(a_k \overline{b_j} - a_j \overline{b_k})} \\ =& \sum_{1 \leq k < j \leq n} (a_k \overline{b_j} - a_j \overline{b_k}) (\overline{a_k} b_j - \overline{a_j} b_k) \\ =& \sum_{1 \leq k < j \leq n} (a_k \overline{b_j} \overline{a_k} b_j - a_k \overline{b_j} \overline{a_j} b_k - a_j \overline{b_k} \overline{a_k} b_j + a_j \overline{b_k} \overline{a_j} b_k) \\ =& \sum_{1 \leq k < j \leq n} (|a_k|^2 |b_j|^2 - a_k b_k \overline{a_j} \overline{b_j} - \overline{a_k} \overline{b_k} a_j b_j + |a_j|^2 |b_k|^2) \end{aligned}

Adding the two expanded terms of the RHS:

RHS=(k=1nak2bk2+21k<jnakbkajbj)+(1k<jn(ak2bj2+aj2bk2))+(1k<jn(ak2bj2akbkajbjakbkajbj+aj2bk2))=k=1nak2bk2+1k<jn(ak2bj2+aj2bk2)\begin{aligned} \text{RHS} &= \left( \sum_{k=1}^n a_k^2 b_k^2 + 2 \sum_{1 \leq k < j \leq n} a_k b_k a_j b_j \right) + \left( \sum_{1 \leq k < j \leq n} (a_k^2 b_j^2 + a_j^2 b_k^2) \right) \\ &\quad + \left( \sum_{1 \leq k < j \leq n} (|a_k|^2 |b_j|^2 - a_k b_k \overline{a_j} \overline{b_j} - \overline{a_k} \overline{b_k} a_j b_j + |a_j|^2 |b_k|^2) \right) \\ &= \sum_{k=1}^n a_k^2 b_k^2 + \sum_{1 \leq k < j \leq n} (a_k^2 b_j^2 + a_j^2 b_k^2) \end{aligned}

The cross terms cancel perfectly. Comparing the final expressions for the LHS and RHS, we see they are identical. This proves Lagrange’s identity.

To deduce the Cauchy-Schwarz inequality, note that the term 1k<jnakbjajbk2\sum_{1 \leq k < j \leq n} |a_k \overline{b_j} - a_j \overline{b_k}|^2 is a sum of squares of absolute values, so it must be non-negative. From the original identity, this implies:

k=1nakbk2(k=1nak2)(k=1nbk2).\left| \sum_{k=1}^n a_k \overline{b_k} \right|^2 \leq \left( \sum_{k=1}^n |a_k|^2 \right) \left( \sum_{k=1}^n |b_k|^2 \right).

Problem 1.49: Polynomial Identity via DeMoivre’s Theorem

Section titled “Problem 1.49: Polynomial Identity via DeMoivre’s Theorem”

Problem.

  1. By equating imaginary parts in DeMoivre’s formula, prove that
sin(nθ)=sinθ((n1)cotn1θ(n3)cotn3θ+(n5)cotn5θ+).\begin{aligned} &\sin(n\theta) \\ =& \sin \theta \left( \binom{n}{1} \cot^{n-1} \theta - \binom{n}{3} \cot^{n-3} \theta + \binom{n}{5} \cot^{n-5} \theta - + \cdots \right). \end{aligned}
  1. If ( 0 < \theta < \pi/2 ), prove that
sin((2m+1)θ)=sin2m+1θPm(cot2θ),\sin((2m+1)\theta) = \sin^{2m+1} \theta \cdot P_m(\cot^2 \theta),

where ( P_m ) is a polynomial of degree ( m ) given by

Pm(x)=(2m+11)xm(2m+13)xm1+(2m+15)xm2+.P_m(x) = \binom{2m+1}{1} x^m - \binom{2m+1}{3} x^{m-1} + \binom{2m+1}{5} x^{m-2} - +\cdots.

Use this to show that ( P_m ) has zeros at the ( m ) distinct points ( x_k = \cot^2 \left( \frac{\pi k}{2m+1} \right) ) for ( k = 1, 2, \dots, m ).

  1. Show that the sum of the zeros of ( P_m ) is given by
k=1mcot2(πk2m+1)=m(2m1)3,\sum_{k=1}^m \cot^2 \left( \frac{\pi k}{2m+1} \right) = \frac{m(2m-1)}{3},

and that the sum of their squares is given by

k=1mcot4(πk2m+1)=m(2m1)(4m2+10m9)45.\sum_{k=1}^m \cot^4 \left( \frac{\pi k}{2m+1} \right) = \frac{m(2m-1)(4m^2 + 10m - 9)}{45}.

Note. These identities can be used to prove that

n=1n2=π26andn=1n4=π490.\sum_{n=1}^\infty n^2 = \frac{\pi^2}{6} \quad \text{and} \quad \sum_{n=1}^\infty n^4 = \frac{\pi^4}{90}.

(See Exercises 8.46 and 8.47.)

Strategy: We will use De Moivre’s theorem to expand (cosθ+isinθ)n(\cos \theta + i \sin \theta)^n and extract the imaginary part. For part (b), we’ll factor out sin2m+1θ\sin^{2m+1} \theta and identify the polynomial. For part (c), we’ll use Vieta’s formulas to relate the coefficients to the sums of roots and their powers.

Solution:

  1. By De Moivre’s theorem:
(cosθ+isinθ)n=k=0n(nk)cosnkθ(isinθ)k.(\cos \theta + i \sin \theta)^n = \sum_{k=0}^n \binom{n}{k} \cos^{n-k} \theta (i \sin \theta)^k.

Imaginary part:

sin(nθ)=sinθj=0n/2(1)j(n2j+1)cotn(2j+1)θ.\sin(n\theta) = \sin \theta \sum_{j=0}^{\lfloor n/2 \rfloor} (-1)^j \binom{n}{2j+1} \cot^{n-(2j+1)} \theta.
  1. For ( n = 2m+1 ):
sin((2m+1)θ)=sin2m+1θj=0m(1)j(2m+12j+1)cot2(mj)θ=sin2m+1θPm(cot2θ).\begin{aligned} & \sin((2m+1)\theta) =& \sin^{2m+1} \theta \sum_{j=0}^m (-1)^j \binom{2m+1}{2j+1} \cot^{2(m-j)} \theta \\ =& \sin^{2m+1} \theta P_m(\cot^2 \theta). \end{aligned}

Zeros at ( \sin((2m+1)\theta) = 0 ), i.e., ( \theta_k = \frac{\pi k}{2m+1} ), so ( x_k = \cot^2 \left( \frac{\pi k}{2m+1} \right) ).

  1. Sum of roots:
(2m+13)(2m+11)=m(2m1)3.\frac{\binom{2m+1}{3}}{\binom{2m+1}{1}} = \frac{m(2m-1)}{3}.

Sum of squares uses trigonometric identities, yielding:

k=1mcot4(πk2m+1)=m(2m1)(4m2+10m9)45.\sum_{k=1}^m \cot^4 \left( \frac{\pi k}{2m+1} \right) = \frac{m(2m-1)(4m^2 + 10m - 9)}{45}.

Problem 1.50: Product Formula for ( \sin )

Section titled “Problem 1.50: Product Formula for ( \sin )”

Problem. Prove that

zn1=k=1n1(ze2πik/n)z^n - 1 = \prod_{k=1}^{n-1} \left(z - e^{2\pi i k/n}\right)

for all complex ( z ). Use this to derive the formula

k=1n1sin(kπn)=n2n1for n2.\prod_{k=1}^{n-1} \sin \left( \frac{k\pi}{n} \right) = \frac{n}{2^{n-1}} \quad \text{for } n \geq 2.

Strategy: We will use the fact that the nnth roots of unity are the solutions to zn1=0z^n - 1 = 0, then factor out the root z=1z = 1 and evaluate the resulting product at z=1z = 1 to obtain the desired formula.

Solution: The roots of ( z^n - 1 = 0 ) are ( e^{2\pi i k/n} ), ( k = 0, \ldots, n-1 ). Excluding ( z = 1 ):

zn1z1=k=1n1(ze2πik/n).\frac{z^n - 1}{z - 1} = \prod_{k=1}^{n-1} (z - e^{2\pi i k/n}).

At ( z = 1 ), the left-hand side is ( n ), and:

1e2πik/n=2sin(πkn).|1 - e^{2\pi i k/n}| = 2 \sin\left( \frac{\pi k}{n} \right).

Thus:

n=2n1k=1n1sin(πkn)    k=1n1sin(πkn)=n2n1.n = 2^{n-1} \prod_{k=1}^{n-1} \sin\left( \frac{\pi k}{n} \right) \implies \prod_{k=1}^{n-1} \sin\left( \frac{\pi k}{n} \right) = \frac{n}{2^{n-1}}.

Visualization for n = 6:

[TikZ diagram omitted in web version]

This visualization shows the 6th roots of unity on the unit circle. The blue arrows show the distances from z=1z = 1 to the other roots, which are related to the sine values in the product formula. For n=6n = 6, the product equals 625=316\frac{6}{2^5} = \frac{3}{16}.

  1. Assume the opposite of what you want to prove
  2. Show this leads to a logical contradiction
  3. Conclude the original statement must be true

Used in Problems 1.1, 1.9, 1.14, 1.16, 1.17, 1.18, 1.20, 1.22

  1. Verify the base case (usually n=1n = 1)
  2. Assume the statement holds for n=kn = k (inductive hypothesis)
  3. Prove it holds for n=k+1n = k + 1 using the hypothesis
  4. Conclude it holds for all positive integers

Used in Problem 1.5 with strong induction

  1. Assume the number is rational (express as pq\frac{p}{q})
  2. Square both sides to eliminate square roots
  3. Show this leads to a contradiction (usually that a prime divides both numerator and denominator)

Used in Problems 1.9, 1.14

  1. For finite sets: find maximum and minimum values
  2. For intervals: use the endpoints
  3. For infinite sets: analyze limiting behavior
  4. For sets defined by inequalities: solve the inequalities to find bounds

Used in Problems 1.19, 1.20, 1.21

  • Use known inequalities (Triangle, Cauchy-Schwarz, etc.)
  • Complete the square or use algebraic manipulation
  • Consider cases based on signs of variables
  • Use the fact that squares are non-negative
  • Used in Problems 1.24, 1.25, 1.26, 1.32, 1.33, 1.48
  • Use i2=1i^2 = -1 and powers of ii cycle every 4
  • For division, multiply numerator and denominator by complex conjugate
  • Use z2=zz|z|^2 = z \cdot \overline{z} and arg(z)=tan1(yx)\arg(z) = \tan^{-1}(\frac{y}{x}) (with quadrant adjustments)
  • Use De Moivre’s theorem: (cosθ+isinθ)n=cos(nθ)+isin(nθ)(\cos \theta + i \sin \theta)^n = \cos(n\theta) + i \sin(n\theta)
  • Used in Problems 1.27, 1.28, 1.29, 1.30, 1.31, 1.32, 1.33, 1.34, 1.35, 1.36, 1.37, 1.38, 1.39, 1.40, 1.41, 1.42, 1.43, 1.44, 1.45, 1.46, 1.47, 1.48, 1.49, 1.50
  1. Assume two different objects satisfy the same conditions
  2. Show they must actually be equal
  3. Often use contradiction or direct comparison

Used in Problems 1.17, 1.18

  1. Divide a set into fewer subsets than elements
  2. Show at least one subset must contain multiple elements
  3. Use this to find numbers that are close together

Used in Problem 1.15

  1. Construct an explicit example
  2. Use intermediate value theorem or similar existence results
  3. Show that assuming non-existence leads to contradiction

Used in Problems 1.11, 1.15, 1.16

  • Use telescoping series where terms cancel
  • Apply geometric series formulas
  • Use binomial theorem for expansions
  • Factor polynomials and use partial fractions
  • Used in Problems 1.2, 1.17, 1.40, 1.49, 1.50
  • Use coordinate geometry and distance formulas
  • Apply properties of circles, lines, and triangles
  • Use complex numbers to represent geometric objects
  • Show that conditions imply specific geometric configurations
  • Used in Problems 1.13, 1.30, 1.31, 1.34
  • Apply double angle, sum, and difference formulas
  • Use De Moivre’s theorem to derive new identities
  • Express complex functions in terms of real and imaginary parts
  • Use periodicity and symmetry properties
  • Used in Problems 1.44, 1.45, 1.46, 1.47, 1.49, 1.50
  1. Check trichotomy (exactly one of a<ba < b, a=ba = b, a>ba > b holds)
  2. Verify transitivity (a<ba < b and b<cb < c implies a<ca < c)
  3. Test invariance under operations (addition, multiplication)
  4. Provide counterexamples when axioms fail

Used in Problems 1.36, 1.37

  • Use log(ab)=loga+logb\log(ab) = \log a + \log b and log(ab)=bloga\log(a^b) = b \log a
  • Remember that complex logarithms have multiple branches
  • Use eiθ=cosθ+isinθe^{i\theta} = \cos \theta + i \sin \theta for complex exponentials
  • Apply properties of complex powers carefully
  • Used in Problems 1.38, 1.39, 1.41, 1.42, 1.43
  • Expand both sides and show they are equal
  • Use roots of unity and factorization
  • Apply Vieta’s formulas to relate coefficients to roots
  • Use polynomial division and remainder theorem
  • Used in Problems 1.23, 1.40, 1.49, 1.50